Параллельное и последовательное соединение формулы: Последовательное и параллельное соединение проводников

alexxlab | 25.11.1981 | 0 | Разное

Содержание

Последовательное и параллельное соединение проводников

Последовательное и параллельное соединение очень широко используется в электронике и электротехнике и порой даже необходимо для правильной работы того или иного узла электроники. И начнем, пожалуй, с самых простых компонентов радиоэлектронных цепей — проводников.

Для начала давайте вспомним, что такое проводник? Проводник — это вещество или какой-либо материал, который отлично проводит электрический ток. Если какой-либо проводник отлично проводит электрический ток, то он в любом случае обладает каким-либо сопротивлением. Сопротивление проводника мы находим по формуле:

формула сопротивление проводника

ρ – это удельное сопротивление, Ом × м

R – сопротивление проводника, Ом

S – площадь поперечного сечения, м2

l – длина проводника, м

Более подробно об этом я писал здесь.

Следовательно, любой проводник представляет из себя резистор с каким-либо сопротивлением. Значит, любой проводник можно нарисовать так.

обозначение резистора на схемах

Последовательное соединение проводников

Сопротивление при последовательном соединении проводников

Последовательное соединение проводников — это когда к одному проводнику мы соединяем другой проводник и так по цепочке. Это и есть последовательное соединение проводников. Их можно соединять с друг другом сколь угодно много.

последовательное соединение резисторов

Чему же будет равняться их общее сопротивление? Оказывается, все просто. Оно будет равняться сумме всех сопротивлений проводников в этой цепи.

Получается, можно записать, что

формула при последовательном соединении резисторов

Пример

У нас есть 3 проводника, которые соединены последовательно. Сопротивление первого 3 Ома, второго 5 Ом, третьего 2 Ома. Найти их общее сопротивление в цепи.

Решение

Rобщее =R1 + R2 + R3 = 3+5+2=10 Ом.

То есть, как вы видите, цепочку из 3 резисторов мы просто заменили на один резистор R

AB .

показать на реальном примере с помощью мультиметра
Видео где подробно расписывается про эти соединения:

Сила тока через последовательное соединение проводников

Что будет, если мы подадим напряжение на концы такого резистора? Через него сражу же побежит электрический ток, сила которого будет вычисляться по закону Ома I=U/R.

Получается, если через резистор RAB течет какой-то определенный ток, следовательно, если разложить наш резистор на составляющие R1 , R2 , R3 , то получится, что через них течет та же самая сила тока, которая текла через резистор RAB .

сила тока через последовательное соединение проводников

Получается, что при последовательном соединении проводников сила тока, которая течет через каждый проводник одинакова. То есть через резистор R1 течет такая же сила тока, как и через резистор R2 и такая же сила тока течет через резистор R3 .

Напряжение при последовательном соединении проводников

Давайте еще раз рассмотрим цепь с тремя резисторами

Как мы уже знаем, при последовательном соединении через каждый резистор проходит одна и та же сила тока. Но вот что будет с напряжением на каждом резисторе и как его найти?

Оказывается, все довольно таки просто. Для этого надо снова вспомнить закон дядюшки Ома и просто вычислить напряжение на  любом резисторе. Давайте так и сделаем.

Пусть у нас будет цепь с такими параметрами.

Мы теперь знаем, что сила тока в такой цепи будет везде одинакова. Но какой ее номинал? Вот в чем загвоздка. Для начала нам надо привести эту цепь к такому виду.

Получается, что в данном случае R

AB =R1 + R2 + R3 = 2+3+5=10 Ом. Отсюда уже находим силу тока по закону Ома I=U/R=10/10=1 Ампер.

Половина дела сделано. Теперь осталось узнать, какое напряжение падает на каждом резисторе. То есть нам надо найти значения UR1 , UR2 , UR3  . Но как это сделать?

Да все также, через закон Ома. Мы знаем, что через каждый резистор проходит сила тока 1 Ампер, мы уже вычислили это значение. Закон ома гласит I=U/R , отсюда получаем, что U=IR.

Следовательно,

UR1 = IR1 =1×2=2 Вольта

UR2 = IR2 = 1×3=3 Вольта

UR3 = IR3 =1×5=5 Вольт

Теперь начинается самое интересное. Если сложить все падения напряжений на резисторах, то можно получить… напряжение источника! Он у нас равен 10 Вольт.

Получается

U=UR1+UR2+UR3

Мы получили самый простой делитель напряжения.

Вывод: сумма падений напряжений при последовательном соединении равняется напряжению питания.

 

Параллельное соединение проводников

Параллельное соединение проводников выглядит вот так.

параллельное соединение резисторов

Ну что, думаю, начнем с сопротивления.

Сопротивление при параллельном соединении проводников

Давайте пометим клеммы как А и В

В этом случае общее сопротивление RAB будет находиться по формуле

 

Если же мы имеем только два параллельно соединенных проводника

То в этом случае можно упростить длинную неудобную формулу и она примет вид такой вид.

Напряжение при параллельном соединении проводников

Здесь, думаю ничего гадать не надо. Так как все проводники соединяются параллельно, то и напряжение у всех будет одинаково.

Получается, что напряжение на R1 будет такое же как и на R2, как и на R3, так и на Rn

Сила тока при параллельном соединении проводников

Если с напряжением все понятно, то с силой тока могут быть небольшие затруднения. Как вы помните, при последовательном соединении сила тока через каждый проводник была одинакова. Здесь же совсем наоборот. Через каждый проводник

будет течь своя сила тока. Как же ее вычислить? Придется опять прибегать к Закону Ома.

Чтобы опять же было нам проще, давайте рассмотрим все это дело на реальном примере. На рисунке ниже видим параллельное соединение трех резисторов, подключенных к источнику питания U.

Как мы уже знаем, на каждом резисторе одно и то же напряжение U. Но будет ли сила тока такая же, как и во всей цепи? Нет. Поэтому для каждого резистора мы должны вычислить свою силу тока по закону Ома I=U/R. В результате получаем, что

I1 = U/R1

I2 = U/R2

I3 = U/R3

Если бы у нас еще были резисторы, соединенные параллельно, то для них

In = U/Rn

В этом случае, сила тока в цепи будет равна:

Задача

Вычислить силу тока через каждый резистор и силу тока в цепи, если известно напряжение источника питания и номиналы резисторов.

Решение

Воспользуемся формулами, которые приводили выше.

I1 = U/R1

I2 = U/R2

I3 = U/R3

Если бы у нас еще были резисторы, соединенные параллельно, то для них

In = U/Rn

Следовательно,

I1 = U/R1 = 10/2=5 Ампер

I2 = U/R2 = 10/5=2 Ампера

I3 = U/R3 = 10/10=1 Ампер

Далее, воспользуемся формулой

чтобы найти силу тока, которая течет в цепи

I=I1 + I2 + I3 = 5+2+1=8 Ампер

2-ой способ найти I

I=U/Rобщее

Чтобы найти Rобщее мы должны воспользоваться формулой

Чтобы не париться с вычислениями, есть онлайн калькуляторы. Вот один из них — «калькулятор резисторов«. Я за вас уже все вычислил. Параллельное соединение 3-ех резисторов номиналом в 2, 5, и 10 Ом равняется 1,25 Ом, то есть Rобщее = 1,25 Ом.

I=U/Rобщее = 10/1,25=8 Ампер.

Параллельное соединение резисторов в электронике также называется делителем тока, так как резисторы делят ток между собой.

Ну а вот вам бонусом объяснение, что такое последовательное и параллельное соединение проводников от лучшего преподавателя России.

Подробное объяснение на видео:

Прикольный набор радиолюбителя по ссылке <<<

Похожие статьи по теме «последовательное и параллельное соединение»

Закон Ома

Проводник (электрический проводник)

Что такое резистор

Делитель напряжения

Делитель тока

Что такое напряжение

Что такое сила тока

Последовательное и параллельное соединение. Применение и схемы

В электрических цепях элементы могут соединяться по различным схемам, в том числе они имеют последовательное и параллельное соединение.

Последовательное соединение

При таком соединении проводники соединяются друг с другом последовательно, то есть, начало одного проводника будет соединяться с концом другого. Основная особенность данного соединения заключается в том, что все проводники принадлежат одному проводу, нет никаких разветвлений. Через каждый из проводников будет протекать один и тот же электрический ток. Но суммарное напряжение на проводниках будет равняться вместе взятым напряжениям на каждом из них.

Рассмотрим некоторое количество резисторов, соединенных последовательно. Так как нет разветвлений, то количество проходящего заряда через один проводник, будет равно количеству заряда, прошедшего через другой проводник. Силы тока на всех проводниках будут одинаковыми. Это основная особенность данного соединения.

Это соединение можно рассмотреть иначе. Все резисторы можно заменить одним эквивалентным резистором.

Ток на эквивалентном резисторе будет совпадать с общим током, протекающим через все резисторы. Эквивалентное общее напряжение будет складываться из напряжений на каждом резисторе. Это является разностью потенциалов на резисторе.

Если воспользоваться этими правилами и законом Ома, который подходит для каждого резистора, можно доказать, что сопротивление эквивалентного общего резистора будет равно сумме сопротивлений. Следствием первых двух правил будет являться третье правило.

Применение

Последовательное соединение используется, когда нужно целенаправленно включать или выключать какой-либо прибор, выключатель соединяют с ним по последовательной схеме. Например, электрический звонок будет звенеть только тогда, когда он будет последовательно соединен с источником и кнопкой. Согласно первому правилу, если электрический ток отсутствует хотя бы на одном из проводников, то его не будет и на других проводниках. И наоборот, если ток имеется хотя бы на одном проводнике, то он будет и на всех других проводниках. Также работает карманный фонарик, в котором есть кнопка, батарейка и лампочка. Все эти элементы необходимо соединить последовательно, так как нужно, чтобы фонарик светил, когда будет нажата кнопка.

Иногда последовательное соединение не приводит к нужным целям. Например, в квартире, где много люстр, лампочек и других устройств, не следует все лампы и устройства соединять последовательно, так как никогда не требуется одновременно включать свет в каждой из комнат квартиры. Для этого последовательное и параллельное соединение рассматривают отдельно, и для подключения осветительных приборов в квартире применяют параллельный вид схемы.

Параллельное соединение

В этом виде схемы все проводники соединяются параллельно друг с другом. Все начала проводников объединены в одну точку, и все концы также соединены вместе. Рассмотрим некоторое количество однородных проводников (резисторов), соединенных по параллельной схеме.

Этот вид соединения является разветвленным. В каждой ветви содержится по одному резистору. Электрический ток, дойдя до точки разветвления, разделяется на каждый резистор, и будет равняться сумме токов на всех сопротивлениях. Напряжение на всех элементах, соединенных параллельно, является одинаковым.

Все резисторы можно заменить одним эквивалентным резистором. Если воспользоваться законом Ома, можно получить выражение сопротивления. Если при последовательном соединении сопротивления складывались, то при параллельном будут складываться величины обратные им, как записано в формуле выше.

Применение

Если рассматривать соединения в бытовых условиях, то в квартире лампы освещения, люстры должны быть соединены параллельно. Если их соединить последовательно, то при включении одной лампочки мы включим все остальные. При параллельном же соединении мы можем, добавляя соответствующий выключатель в каждую из ветвей, включать соответствующую лампочку по мере желания. При этом такое включение одной лампы не влияет на остальные лампы.

Все электрические бытовые устройства в квартире соединены параллельно в сеть с напряжением 220 В, и подключены к распределительному щитку. Другими словами, параллельное соединение используется при необходимости подключения электрических устройств независимо друг от друга. Последовательное и параллельное соединение имеют свои особенности. Существуют также смешанные соединения.

Работа тока

Последовательное и параллельное соединение, рассмотренное ранее, было справедливо для величин напряжения, сопротивления и силы тока, являющихся основными. Работа тока определяется по формуле:

А = I х U х t, где А – работа тока, t – время течения по проводнику.

Для определения работы при последовательной схеме соединения, необходимо заменить в первоначальном выражении напряжение. Получаем:

А=I х (U1 + U2) х t

Раскрываем скобки и получаем, что на всей схеме работа определяется суммой на каждой нагрузке.

Точно также рассматриваем параллельную схему соединения. Только меняем уже не напряжение, а силу тока. Получается результат:

А = А1+А2

Мощность тока

При рассмотрении формулы мощности участка цепи снова необходимо пользоваться формулой:

Р=U х I

После аналогичных рассуждений выходит результат, что последовательное и параллельное соединение можно определить следующей формулой мощности:

Р=Р1 + Р2

Другими словами, при любых схемах общая мощность равна сумме всех мощностей в схеме. Этим можно объяснить, что не рекомендуется включать в квартире сразу несколько мощных электрических устройств, так как проводка может не выдержать такой мощности.

Влияние схемы соединения на новогоднюю гирлянду

После перегорания одной лампы в гирлянде можно определить вид схемы соединения. Если схема последовательная, то не будет гореть ни одной лампочки, так как сгоревшая лампочка разрывает общую цепь. Чтобы выяснить, какая именно лампочка сгорела, нужно проверять все подряд. Далее, заменить неисправную лампу, гирлянда будет функционировать.

При применении параллельной схемы соединения гирлянда будет продолжать работать, даже если одна или несколько ламп сгорели, так как цепь не разорвана полностью, а только один небольшой параллельный участок. Для восстановления такой гирлянды достаточно увидеть, какие лампы не горят, и заменить их.

Последовательное и параллельное соединение для конденсаторов

При последовательной схеме возникает такая картина: заряды от положительного полюса источника питания идут только на наружные пластины крайних конденсаторов. Конденсаторы, находящиеся между ними, передают заряд по цепи. Этим объясняется появление на всех пластинах равных зарядов с разными знаками. Исходя из этого, заряд любого конденсатора, соединенного по последовательной схеме, можно выразить такой формулой:

qобщ= q1 = q2 = q3

Для определения напряжения на любом конденсаторе, необходима формула:

U= q/С

Где С — емкость. Суммарное напряжение выражается таким же законом, который подходит для сопротивлений. Поэтому получаем формулу емкости:

С= q/(U1 + U2 + U3)

Чтобы сделать эту формулу проще, можно перевернуть дроби и заменить отношение разности потенциалов к заряду емкости. В результате получаем:

1/С= 1/С1 + 1/С2 + 1/C3

Немного иначе рассчитывается параллельное соединение конденсаторов.

Общий заряд вычисляется как сумма всех зарядов, накопившихся на пластинах всех конденсаторов. А величина напряжения также вычисляется по общим законам. В связи с этим формула суммарной емкости при параллельной схеме соединения выглядит так:

С= (q1 + q2 + q3)/U

Это значение рассчитывается как сумма каждого прибора в схеме:

С=С1 + С2 + С3

Смешанное соединение проводников

В электрической схеме участки цепи могут иметь и последовательное и параллельное соединение, переплетающихся между собой. Но все законы, рассмотренные выше для отдельных видов соединений, справедливы по-прежнему, и используются по этапам.

Сначала нужно мысленно разложить схему на отдельные части. Для лучшего представления ее рисуют на бумаге. Рассмотрим наш пример по изображенной выше схеме.

Удобнее всего ее изобразить, начиная с точек Б и В. Они расставляются на некотором расстоянии между собой и от края листа бумаги. С левой стороны к точке Б подключается один провод, а справа отходят два провода. Точка В наоборот, слева имеет две ветки, а после точки отходит один провод.

Далее нужно изобразить пространство между точками. По верхнему проводнику расположены 3 сопротивления с условными значениями 2, 3, 4. Снизу будет идти ток с индексом 5. Первые 3 сопротивления включены в схему последовательно, а пятый резистор подключен параллельно.

Остальные два сопротивления (первый и шестой) подключены последовательно с рассматриваемым нами участком Б-В. Поэтому схему дополняем 2-мя прямоугольниками по сторонам от выбранных точек.

Теперь используем формулу расчета сопротивления:
  • Первая формула для последовательного вида соединения.
  • Далее, для параллельной схемы.
  • И окончательно для последовательной схемы.

Аналогичным образом можно разложить на отдельные схемы любую сложную схему, включая соединения не только проводников в виде сопротивлений, но и конденсаторов. Чтобы научиться владеть приемами расчета по разным видам схем, необходимо потренироваться на практике, выполнив несколько заданий.

Похожие темы:

Последовательное и параллельное соединение

В данной статье речь пойдет о последовательном и параллельном соединении проводников. На примерах будут рассмотрены данные соединения и как при таких соединениях будут изменяться такие величины как:

  • ток;
  • напряжение;
  • сопротивление.

В таблице 1.8 [Л2, с.24] приведены схемы и формулы по определению сопротивлений, токов и напряжений при параллельном и последовательном соединении.

Последовательное соединение

Последовательным соединением называются те участки цепи, по которым всегда проходят одинаковые токи.

При последовательном соединении:

  • сила тока во всех проводниках одинакова;
  • напряжение на всём соединении равно сумме напряжений на отдельных проводниках;
  • сопротивление всего соединения равно сумме сопротивлений отдельных проводников.

Пример 1

Последовательно подключены две лампы накаливания одинаковой мощности Рл1=Рл2=100 Вт к сети с напряжением Uн=220В. Сопротивление нити в лампах составляет Rл1=Rл2=122 Ом. Номинальное напряжение для ламп равно 220 B. На рис.1 показано последовательное включение ламп.

Решение

Составляем схему замещения, выражая каждую из входящих элементов цепи (в данном случае лампы накаливания) в виде сопротивлений.

1. Определяем ток протекающей по участкам цепи:

Iн = Uн/Rл1+ Rл2 = 220/122+122 = 0,9 A

2. Определяем напряжение на каждой из ламп накаливания, так как мощность ламп у нас одинаковая, то и напряжение для каждой из ламп будет одинаково:

Uл1=Uл2 = Iн*R = 0,9*122 = 110 B

Как мы видим напряжение источника (в данном примере 220 В) разделиться поровну, между обоими последовательно включенными лампами. При этом лампы будут ели светит, их накал будет неполным.

Для того чтобы лампы горели с полным накалом, нужно увеличить напряжение источника с 220В до 440В, при этом на каждой из ламп установиться номинальное (рабочее) напряжение равное 220В.

Пример 2

Последовательно подключены две лампы накаливания мощность Рл1 = 100 Вт и Рл2 = 75 Вт к сети с напряжением Uн=220В. Сопротивление нити в лампах составляют Rл1= 122 Ом для стоваттной лампы и Rл2= 153 Ом для семидесяти пяти ватной лампы.

Решение

1. Определяем ток протекающей по участкам цепи:

Iн = Uн/Rл1+ Rл2 = 220/100+75 = 0,8 A

2. Определяем напряжение на каждой из ламп накаливания:

Uл1= Iн*Rл1 = 0,8*122 = 98 B
Uл2= Iн*Rл2 = 0,8*153 = 122 B

Исходя из результатов расчетов, более мощная лампа 100 Вт получает при этом меньшее напряжение. Но ток в двух последовательно включенных даже разных лампах остается одинаковым. Например, если одна из ламп перегорит (порвется ее нить накаливания), погаснут обе лампы.

Данное соединение лампочек, например, используется в трамвайном вагоне для освещения салона.

Параллельное соединение

Параллельное соединение – это соединение, при котором начала всех проводников присоединяются к одной точке цепи, а их концы к другой.

Точки цепи, к которым сходится несколько проводов, называют узлами. Участки цепи, соединяющие между собой узлы, называют ветвями.

При параллельном соединении:

  • напряжение на всех проводниках одинаково;
  • сила тока в месте соединения проводников равна сумме токов в отдельных проводниках;
  • величина, обратная сопротивлению всего соединения, равна сумме величин, обратных сопротивлениям отдельных проводников.

Пример 3

Определить токи и напряжения всех участков цепи (рис.5), если известно:

  • Номинальное напряжение сети Uн = 220В;
  • Сопротивление нити в лампах HL1 и HL2 составляют Rл1 = Rл2 = 122 Ом.
  • Сопротивление нити в лампе HL3 составляют Rл3 = 153 Ом.

Решение

Составляем схему замещения для схемы, представленной на рис.5.

1. Определяем проводимость всей цепи [Л1, с.47] и согласно таблицы 1.8:

2. Определяем сопротивление всей цепи [Л1, с.47]:

3. Определяем силу тока цепи по закону Ома:

4. Определяем токи для каждой цепи [Л1, с.47]:

5. Выполним проверку, согласно которой, сила тока в месте соединения проводников равна сумме токов в отдельных проводниках:

Iл1+ Iл2+ Iл3=Iобщ.=1,8+1,8+1,44=5,04=5,04 (условие выполняется)

Смешанное соединение

Смешанным соединением – называется последовательно-параллельное соединение сопротивлений или участков цепи.

Пример 4

Определить токи и напряжения всех участков цепи (рис.7), если известно:

  • Номинальное напряжение сети Uн = 220В;
  • Сопротивление нити в лампах HL1, HL2, HL3 составляют Rл1 = Rл2 = Rл3 = 122 Ом.
  • Сопротивление нити в лампе HL4 составляют Rл4 = 153 Ом.
  • Результаты расчетов для участка цепи ВС (параллельное соединение проводников) применим из примера 3:
    Сопротивление цепи ВС составляет Rвс = 43,668 Ом.

Решение

Составляем схему замещения для схемы, представленной на рис.7.

1. Определяем сопротивление всей цепи:

Rобщ = Rав+Rвс = Rл1+Rвс = 122+43,688 = 165,688 Ом

2. Определяем силу тока цепи, согласно закона Ома:

3. Определяем напряжение на первом сопротивлении:

Uав=Uл1= Iобщ*Rл1 = 1,33*122 = 162 B

4. Определяем напряжение на участке ВС:

Uвс= Iобщ*Rвс = 1,33*43,688 = 58,1 B

5. Определяем токи для каждой цепи участка ВС:

6. Выполним проверку для участка цепи ВС:

Iл2+ Iл3+ Iл4= Iобщ.=0,48+0,48+0,38=1,33=1,33 (условие выполняется)

Литература:

  1. Общая электротехника с основами электроники, В.С. Попов, 1972 г.
  2. Справочная книга электрика. В.И. Григорьева. 2004 г.

Всего наилучшего! До новых встреч на сайте Raschet.info.

Сила тока при последовательном соединении

В электрических цепях используются различные типы соединений. Основными являются последовательные, параллельные и смешанные схемы подключений. В первом случае используется несколько сопротивлений, соединенных в единую цепочку друг за другом. То есть, начало одного резистора соединяется с концом второго, а начало второго – с концом третьего и так далее, до любого количества сопротивлений. Сила тока при последовательном соединении будет одинаковой во всех точках и на всех участках. Для определения и сравнения других параметров электрической цепи, следует рассматривать и остальные виды соединений, обладающие собственными свойствами и характеристиками.

Последовательное и параллельное соединение сопротивлений

Любая нагрузка обладает сопротивлением, препятствующим свободному течению электрического тока. Его путь проходит от источника тока, через проводники к нагрузке. Для нормального прохождения тока, проводник должен обладать хорошей проводимостью и легко отдавать электроны. Это положение пригодится далее при рассмотрении вопроса, что такое последовательное соединение.

В большинстве электрических цепей применяются медные проводники. Каждая цепь содержит приемники энергии – нагрузки, обладающие различными сопротивлениями. Параметры соединения лучше всего рассматривать на примере внешней цепи источника тока, состоящей из трех резисторов R1, R2, R3. Последовательное соединение предполагает поочередное включение этих элементов в замкнутую цепь. То есть начало R1 соединяется с концом R2, а начало R2 – с концом R3 и так далее. В такой цепочке может быть любое количество резисторов. Эти символы используют в расчетах последовательные и параллельные соединения.

Сила тока на всех участках будет одинаковой: I = I1 = I2 = I3, а общее сопротивление цепи составит сумму сопротивлений всех нагрузок: R = R1 + R2 + R3. Остается лишь определить, каким будет напряжение при последовательном соединении. В соответствии с законом Ома, напряжение представляет собой силу тока и сопротивления: U = IR. Отсюда следует, что напряжение на источнике тока будет равно сумме напряжений на каждой нагрузке, поскольку ток везде одинаковый: U = U1 + U2 + U3.

При постоянном значении напряжения, ток при последовательном соединении будет находиться в зависимости от сопротивления цепи. Поэтому при изменении сопротивления хотя-бы на одной из нагрузок, произойдет изменение сопротивления во всей цепи. Кроме того, изменятся ток и напряжение на каждой нагрузке. Основным недостатком последовательного соединения считается прекращение работы всех элементов цепи, при выходе из строя даже одного из них.

Совершенно другие характеристики тока, напряжения и сопротивления получаются при использовании параллельного соединения. В этом случае начала и концы нагрузок соединяются в двух общих точках. Происходит своеобразное разветвление тока, что приводит к снижению общего сопротивления и росту общей проводимости электрической цепи.

Для того чтобы отобразить эти свойства, вновь понадобится закон Ома. В данном случае сила тока при параллельном соединении и его формула будет выглядеть так: I = U/R. Таким образом, при параллельном соединении n-го количества одинаковых резисторов, общее сопротивление цепи будет в n раз меньше любого из них: Rобщ = R/n. Это указывает на обратно пропорциональное распределение токов в нагрузках по отношению к сопротивлениям этих нагрузок. То есть, при увеличении параллельно включенных сопротивлений, сила тока в них будет пропорционально уменьшаться. В виде формул все характеристики отображаются следующим образом: сила тока – I = I1 + I2 + I3, напряжение – U = U1 = U2 = U3, сопротивление – 1/R = 1/R1 + 1/R2 + 1/R3.

При неизменном значении напряжения между элементами, токи в этих резисторах не имеют зависимости друг от друга. Если один или несколько резисторов будут выключены из цепи, это никак не повлияет на работу других устройств, остающихся включенными. Данный фактор является основным преимуществом параллельного соединения электроприборов.

В схемах обычно не используется только последовательное соединение и параллельное соединение сопротивлений, они применяются в комбинированном виде, известном как смешанное соединение. Для вычисления характеристик таких цепей применяются формулы обоих вариантов. Все расчеты разбиваются на несколько этапов, когда вначале определяются параметры отдельных участков, после чего они складываются и получается общий результат.

Законы последовательного и параллельного соединения проводников

Основным законом, применяемым при расчетах различных видов соединений, является закон Ома. Его основным положением является наличие на участке цепи силы тока, прямо пропорциональной напряжению и обратно пропорциональной сопротивлению на данном участке. В виде формулы этот закон выглядит так: I = U/R. Он служит основой для проведения расчетов электрических цепей, соединяемых последовательно или параллельно. Порядок вычислений и зависимость всех параметров от закона Ома наглядно показаны на рисунке. Отсюда выводится и формула последовательного соединения.

Более сложные вычисления с участием других величин требуют применения правила Кирхгофа. Его основное положение заключается в том, что несколько последовательно соединенных источников тока, будут обладать электродвижущей силой (ЭДС), составляющей алгебраическую сумму ЭДС каждого из них. Общее сопротивление этих батарей будет состоять из суммы сопротивлений каждой батареи. Если выполняется параллельное подключение n-го количества источников с равными ЭДС и внутренними сопротивлениями, то общая сумма ЭДС будет равно ЭДС на любом из источников. Значение внутреннего сопротивления составит rв = r/n. Эти положения актуальны не только для источников тока, но и для проводников, в том числе и формулы параллельное соединение проводников.

В том случае, когда ЭДС источников будет иметь разное значение, для расчетов силы тока на различных участках цепи применяются дополнительные правила Кирхгофа.

теория, формулы, подключение и расчет силы тока

Практически каждому, кто занимался электрикой, приходилось решать вопрос параллельного и последовательного соединения элементов схемы. Некоторые решают проблемы параллельного и последовательного соединения проводников методом «тыка», для многих «несгораемая» гирлянда является необъяснимой, но привычной аксиомой. Тем не менее, все эти и многие другие подобные вопросы легко решаются методом, предложенным еще в самом начале XIX века немецким физиком Георгом Омом. Законы, открытые им, действуют и поныне, а понять их сможет практически каждый.

Основные электрические величины цепи

Для того чтобы выяснить, как то или иное соединение проводников повлияет на характеристики схемы, необходимо определиться с величинами, которые характеризуют любую электрическую цепь. Вот основные из них:

  • Электрическое напряжение, согласно научному определению, это разность потенциалов между двумя точками электрической цепи. Измеряется в вольтах (В). Между клеммами бытовой розетки, к примеру, оно равно 220 В, на батарейке вольтметр покажет 1,5 В, а зарядное устройство вашего планшета или смартфона выдает 5 В. Напряжение бывает переменным и постоянным, но в нашем случае это несущественно.
  • Электрический ток – упорядоченное движение электронов в электрической цепи. Ближайшая аналогия – ток воды в трубопроводе. Измеряется в амперах (А). Если цепь не замкнута, ток существовать не может.
  • Электрическое сопротивление. Величина измеряется в омах (Ом) и характеризует способность проводника или электрической цепи сопротивляться прохождению электрического тока. Если продолжить аналогию с водопроводом, то новая гладкая труба будет иметь маленькое сопротивление, забитая ржавчиной и шлаками – высокое.
  • Электрическая мощность. Эта величина характеризует скорость преобразования электрической энергии в любую другую и измеряется в ваттах (Вт). Кипятильник в 1000 Вт вскипятит воду быстрее стоваттного, мощная лампа светит ярче и т.д.

Взаимная зависимость электрических величин

Теперь необходимо определиться, как все вышеперечисленные величины зависят одна от другой. Правила зависимости несложны и сводятся к двум основным формулам:

Здесь I – ток в цепи в амперах, U – напряжение, подводимое к цепи в вольтах, R – сопротивление цепи в омах, P – электрическая мощность цепи в ваттах.

Предположим, перед нами простейшая электрическая цепь, состоящая из источника питания с напряжением U и проводника с сопротивлением R (нагрузки).

Поскольку цепь замкнута, через нее течет ток I. Какой величины он будет? Исходя из вышеприведенной формулы 1, для его вычисления нам нужно знать напряжение, развиваемое источником питания, и сопротивление нагрузки. Если мы возьмем, к примеру, паяльник с сопротивлением спирали 100 Ом и подключим его к осветительной розетке с напряжением 220 В, то ток через паяльник будет составлять:

220 / 100 = 2,2 А.

Какова мощность этого паяльника? Воспользуемся формулой 2:

2,2 * 220 = 484 Вт.

Хороший получился паяльник, мощный, скорее всего, двуручный. Точно так же, оперируя этими двумя формулами и преобразуя их, можно узнать ток через мощность и напряжение, напряжение через ток и сопротивление и т.д. Сколько, к примеру, потребляет лампочка мощностью 60 Вт в вашей настольной лампе:

60 / 220 = 0,27 А или 270 мА.

Сопротивление спирали лампы в рабочем режиме:

220 / 0,27 = 815 Ом.

Схемы с несколькими проводниками

Все рассмотренные выше случаи являются простыми – один источник, одна нагрузка. Но на практике нагрузок может быть несколько, и соединены они бывают тоже по-разному. Существует три типа соединения нагрузки:

  1. Параллельное.
  2. Последовательное.
  3. Смешанное.

Параллельное соединение проводников

В люстре 3 лампы, каждая по 60 Вт. Сколько потребляет люстра? Верно, 180 Вт. Быстренько подсчитываем сначала ток через люстру:

180 / 220 = 0,818 А.

А затем и ее сопротивление:

220 / 0,818 = 269 Ом.

Перед этим мы вычисляли сопротивление одной лампы (815 Ом) и ток через нее (270 мА). Сопротивление же люстры оказалось втрое ниже, а ток – втрое выше. А теперь пора взглянуть на схему трехрожкового светильника.

Схема люстры с тремя лампами

Все лампы в нем соединены параллельно и подключены к сети. Получается, при параллельном соединении трех ламп общее сопротивление нагрузки уменьшилось втрое? В нашем случае – да, но он частный – все лампы имеют одинаковые сопротивление и мощность. Если каждая из нагрузок будет иметь свое сопротивление, то для подсчета общего значения простого деления на количество нагрузок мало. Но и тут есть выход из положения – достаточно воспользоваться вот этой формулой:

1/Rобщ. = 1/R1 + 1/R2 + … 1/Rn.

Для удобства использования формулу можно легко преобразовать:

Rобщ. = (R1*R2*… Rn) / (R1+R2+ … Rn).

Здесь Rобщ. – общее сопротивление цепи при параллельном включении нагрузки. R1 … Rn – сопротивления каждой нагрузки.

Почему увеличился ток, когда вы включили параллельно три лампы вместо одной, понять несложно – ведь он зависит от напряжения (оно осталось неизменным), деленного на сопротивление (оно уменьшилось). Очевидно, что и мощность при параллельном соединении увеличится пропорционально увеличению тока.

Последовательное соединение

Теперь настала пора выяснить, как изменятся параметры цепи, если проводники (в нашем случае лампы) соединить последовательно.

Последовательно соединенная нагрузка

Расчет сопротивления при последовательном соединении проводников исключительно прост:

Rобщ. = R1 + R2.

Те же три шестидесятиваттные лампы, соединенные последовательно, составят уже 2445 Ом (см. расчеты выше). Какими будут последствия увеличения сопротивления цепи? Согласно формулам 1 и 2 становится вполне понятно, что мощность и сила тока при последовательном соединении проводников упадет. Но почему теперь все лампы горят тускло? Это одно из самых интересных свойств последовательного подключения проводников, которое очень широко используется. Взглянем на гирлянду из трех знакомых нам, но последовательно соединенных ламп.

Последовательное соединение трех ламп в гирлянду

Общее напряжение, приложенное ко всей цепи, так и осталось 220 В. Но оно поделилось между каждой из ламп пропорционально их сопротивлению! Поскольку лампы у нас одинаковой мощности и сопротивления, то напряжение поделилось поровну: U1 = U2 = U3 = U/3. То есть на каждую из ламп подается теперь втрое меньшее напряжение, вот почему они светятся так тускло. Возьмете больше ламп – яркость их упадет еще больше. Как рассчитать падение напряжения на каждой из ламп, если все они имеют различные сопротивления? Для этого достаточно четырех формул, приведенных выше. Алгоритм расчета будет следующим:

  1. Измеряете сопротивление каждой из ламп.
  2. Рассчитываете общее сопротивление цепи.
  3. По общим напряжению и сопротивлению рассчитываете ток в цепи.
  4. По общему току и сопротивлению ламп вычисляете падение напряжения на каждой из них.

Хотите закрепить полученные знания? Решите простую задачу, не заглядывая в ответ в конце:

В вашем распоряжении есть 15 однотипных миниатюрных лампочек, рассчитанных на напряжение 13,5 В. Можно ли из них сделать елочную гирлянду, подключаемую к обычной розетке, и если можно, то как?

Смешанное соединение

С параллельным и последовательным соединением проводников вы, конечно, без труда разобрались. Но как быть, если перед вами оказалась примерно такая схема?

Смешанное соединение проводников

Как определить общее сопротивление цепи? Для этого вам понадобится разбить схему на несколько участков. Вышеприведенная конструкция достаточно проста и участков будет два – R1 и R2,R3. Сначала вы рассчитываете общее сопротивление параллельно соединенных элементов R2,R3 и находите Rобщ.23. Затем вычисляете общее сопротивление всей цепи, состоящей из R1 и Rобщ.23, соединенных последовательно:

  • Rобщ.23 = (R2*R3) / (R2+R3).
  • Rцепи = R1 + Rобщ.23.

Задача решена, все очень просто. А теперь вопрос несколько сложнее.

Сложное смешанное соединение сопротивлений

Как быть тут? Точно так же, просто нужно проявить некоторую фантазию. Резисторы R2, R4, R5 соединены последовательно. Рассчитываем их общее сопротивление:

Rобщ.245 = R2+R4+R5.

Теперь параллельно к Rобщ.245 подключаем R3:

Rобщ.2345 = (R3* Rобщ.245) / (R3+ Rобщ.245).

Ну а дальше все очевидно, поскольку остались R1, R6 и найденное нами Rобщ.2345, соединенные последовательно:

Rцепи = R1+ Rобщ.2345+R6.

Вот и все!

Ответ на задачу о елочной гирлянде

Лампы имеют рабочее напряжение всего 13.5 В, а в розетке 220 В, поэтому их нужно включать последовательно.

Поскольку лампы однотипные, напряжение сети разделится между ними поровну и на каждой лампочке окажется 220 / 15 = 14,6 В. Лампы рассчитаны на напряжение 13,5 В, поэтому такая гирлянда хоть и заработает, но очень быстро перегорит. Чтобы реализовать задумку, вам понадобится минимум 220 / 13,5 = 17, а лучше 18-19 лампочек.

Схема елочной гирлянды из миниатюрных ламп накаливания

Соединения проводников – материалы для подготовки к ЕГЭ по Физике

Автор статьи — профессиональный репетитор, автор учебных пособий для подготовки к ЕГЭ Игорь Вячеславович Яковлев

Темы кодификатора ЕГЭ: параллельное и последовательное соединение проводников, смешанное соединение проводников.

Есть два основных способа соединения проводников друг с другом — это последовательное и параллельное соединения. Различные комбинации последовательного и параллельного соединений приводят к смешанному соединению проводников.

Мы будем изучать свойства этих соединений, но сначала нам понадобится некоторая вводная информация.

Проводник, обладающий сопротивлением , мы называем резистором и изображаем следующим образом (рис. 1):

Рис. 1. Резистор

Напряжение на резисторе — это разность потенциалов стационарного электрического поля между концами резистора. Между какими именно концами? В общем-то, это неважно, но обычно удобно согласовывать разность потенциалов с направлением тока.

Ток в цепи течёт от «плюса» источника к «минусу». В этом направлении потенциал стационарного поля убывает. Напомним ещё раз, почему это так.

Пусть положительный заряд перемещается по цепи из точки в точку , проходя через резистор (рис. 2):

Рис. 2.

Стационарное поле совершает при этом положительную работу .

Так как и , то и , т. е. .

Поэтому напряжение на резисторе мы вычисляем как разность потенциалов в направлении тока: .

Сопротивление подводящих проводов обычно пренебрежимо мало; на электрических схемах оно считается равным нулю. Из закона Ома следует тогда, что потенциал не меняется вдоль провода: ведь если и , то . (рис. 3):

Рис. 3.

Таким образом, при рассмотрении электрических цепей мы пользуемся идеализацией, которая сильно упрощает их изучение. А именно, мы считаем, что потенциал стационарного поля изменяется лишь при переходе через отдельные элементы цепи, а вдоль каждого соединительного провода остаётся неизменным. В реальных цепях потенциал монотонно убывает при движении от положительной клеммы источника к отрицательной.

Последовательное соединение

При последовательном соединении проводников конец каждого проводника соединяется с началом следующего за ним проводника.

Рассмотрим два резистора и , соединённых последовательно и подключённых к источнику постоянного напряжения (рис. 4). Напомним, что положительная клемма источника обозначается более длинной чертой, так что ток в данной схеме течёт по часовой стрелке.

Рис. 4. Последовательное соединение

Сформулируем основные свойства последовательного соединения и проиллюстрируем их на этом простом примере.

1. При последовательном соединении проводников сила тока в них одинакова.
В самом деле, через любое поперечное сечение любого проводника за одну секунду будет проходить один и тот же заряд. Ведь заряды нигде не накапливаются, из цепи наружу не уходят и не поступают в цепь извне.

2. Напряжение на участке, состоящем из последовательно соединённых проводников, равно сумме напряжений на каждом проводнике.

Действительно, напряжение на участке — это работа поля по переносу единичного заряда из точки в точку ; напряжение на участке — это работа поля по переносу единичного заряда из точки в точку . Складываясь, эти две работы дадут работу поля по переносу единичного заряда из точки в точку , то есть напряжение на всём участке:

Можно и более формально, без всяких словесных объяснений:

3. Сопротивление участка, состоящего из последовательно соединённых проводников, равно сумме сопротивлений каждого проводника.

Пусть — сопротивление участка . По закону Ома имеем:

что и требовалось.

Можно дать интуитивно понятное объяснение правила сложения сопротивлений на одном частном примере. Пусть последовательно соединены два проводника из одинакового вещества и с одинаковой площадью поперечного сечения , но с разными длинами и .

Сопротивления проводников равны:

Эти два проводника образуют единый проводник длиной и сопротивлением

Но это, повторяем, лишь частный пример. Сопротивления будут складываться и в самом общем случае — если различны также вещества проводников и их поперечные сечения.
Доказательство этого даётся с помощью закона Ома, как показано выше.
Наши доказательства свойств последовательного соединения, приведённые для двух проводников, переносятся без существенных изменений на случай произвольного числа проводников.

Параллельное соединение

При параллельном соединении проводников их начала подсоединяются к одной точке цепи, а концы — к другой точке.

Снова рассматриваем два резистора, на сей раз соединённые параллельно (рис. 5).

Рис. 5. Параллельное соединение

Резисторы подсоединены к двум точкам: и . Эти точки называются узлами или точками разветвления цепи. Параллельные участки называются также ветвями; участок от к (по направлению тока) называется неразветвлённой частью цепи.

Теперь сформулируем свойства параллельного соединения и докажем их для изображённого выше случая двух резисторов.

1. Напряжение на каждой ветви одинаково и равно напряжению на неразветвлённой части цепи.
В самом деле, оба напряжения и на резисторах и равны разности потенциалов между точками подключения:

Этот факт служит наиболее отчётливым проявлением потенциальности стационарного электрического поля движущихся зарядов.

2. Сила тока в неразветвлённой части цепи равна сумме сил токов в каждой ветви.
Пусть, например, в точку за время из неразветвлённого участка поступает заряд . За это же время из точки к резистору уходит заряд , а к резистору — заряд .

Ясно, что . В противном случае в точке накапливался бы заряд, меняя потенциал данной точки, что невозможно (ведь ток постоянный, поле движущихся зарядов стационарно, и потенциал каждой точки цепи не меняется со временем). Тогда имеем:

что и требовалось.

3. Величина, обратная сопротивлению участка параллельного соединения, равна сумме величин, обратных сопротивлениям ветвей.
Пусть — сопротивление разветвлённого участка . Напряжение на участке равно ; ток, текущий через этот участок, равен . Поэтому:

Сокращая на , получим:

(1)

что и требовалось.

Как и в случае последовательного соединения, можно дать объяснение данного правила на частном примере, не обращаясь к закону Ома.
Пусть параллельно соединены проводники из одного вещества с одинаковыми длинами , но разными поперечными сечениями и . Тогда это соединение можно рассматривать как проводник той же длины , но с площадью сечения . Имеем:

Приведённые доказательства свойств параллельного соединения без существенных изменений переносятся на случай любого числа проводников.

Из соотношения (1) можно найти :

(2)

К сожалению, в общем случае параллельно соединённых проводников компактного аналога формулы (2) не получается, и приходится довольствоваться соотношением

(3)

Тем не менее, один полезный вывод из формулы (3) сделать можно. Именно, пусть сопротивления всех резисторов одинаковы и равны . Тогда:

откуда

Мы видим, что сопротивление участка из параллельно соединённых одинаковых проводников в раз меньше сопротивления одного проводника.

Смешанное соединение

Смешанное сединение проводников, как следует из названия, может являться совокупностью любых комбинаций последовательного и параллельного соединений, причём в состав этих соединений могут входить как отдельные резисторы, так и более сложные составные участки.

Расчёт смешанного соединения опирается на уже известные свойства последовательного и параллельного соединений. Ничего нового тут уже нет: нужно только аккуратно расчленить данную схему на более простые участки, соединённые последовательно или параллельно.

Рассмотрим пример смешанного соединения проводников (рис. 6).

Рис. 6. Смешанное соединение

Пусть В, Ом, Ом, Ом, Ом, Ом. Найдём силу тока в цепи и в каждом из резисторов.

Наша цепь состоит из двух последовательно соединённых участков и . Сопротивление участка :

Ом.

Участок является параллельным соединением: два последовательно включённых резистора и подключены параллельно к резистору . Тогда:

Ом.

Сопротивление цепи:

Ом.

Теперь находим силу тока в цепи:

A.

Для нахождения тока в каждом резисторе вычислим напряжения на обоих участках:

B;

B.

(Заметим попутно, что сумма этих напряжений равна В, т. е. напряжению в цепи, как и должно быть при последовательном соединении.)

Оба резистора и находятся под напряжением , поэтому:

A;

A.

(В сумме имеем А, как и должно быть при параллельном соединении.)

Сила тока в резисторах и одинакова, так как они соединены последовательно:

А.

Стало быть, через резистор течёт ток A.

Виды соединения проводников

При решении задач принято преобразовывать схему, так, чтобы она была как можно проще. Для этого применяют эквивалентные преобразования. Эквивалентными называют такие преобразования части схемы электрической цепи, при которых токи и напряжения в не преобразованной её части остаются неизменными.

Существует четыре основных вида соединения проводников: последовательное, параллельное, смешанное и мостовое.

Последовательное соединение

Последовательное соединение – это такое соединение, при котором сила тока на всем участке цепи одинакова. Ярким примером последовательного соединения является старая елочная гирлянда. Там лампочки подключены последовательно, друг за другом. Теперь представьте, одна лампочка перегорает, цепь нарушена и остальные лампочки гаснут. Выход из строя одного элемента, ведет за собой отключение всех остальных, это является существенным недостатком последовательного соединения.

При последовательном соединении сопротивления элементов суммируются. 

Параллельное соединение

Параллельное соединение – это соединение, при котором напряжение на концах участка цепи одинаково. Параллельное соединение наиболее распространено, в основном потому, что все элементы находятся под одним напряжением, сила тока распределена по-разному и при выходе одного из элементов все остальные продолжают свою работу.

При параллельном соединении эквивалентное сопротивление находится как:

В случае двух параллельно соединенных резисторов

В случае трех параллельно подключенных резисторов:

Смешанное соединение

Смешанное соединение – соединение, которое является совокупностью последовательных и параллельных соединений. Для нахождения эквивалентного сопротивления нужно, “свернуть” схему поочередным преобразованием параллельных и последовательных участков цепи.

Сначала найдем эквивалентное сопротивление для параллельного участка цепи, а затем прибавим к нему оставшееся сопротивление R3. Следует понимать, что после преобразования эквивалентное сопротивление R1R2 и резистор R3, соединены последовательно.

 

Итак, остается самое интересное и самое сложное соединение проводников.

Мостовая схема

Мостовая схема соединения представлена на рисунке ниже.



Для того чтобы свернуть мостовую схему, один из треугольников моста, заменяют эквивалентной звездой.

И находят сопротивления R1, R2 и R3


Затем находят общее эквивалентное сопротивление, учитывая, что резисторы R3,R4 и R5,R2 соединены между друг другом последовательно, а в парах параллельно. 

На этом всё! Примеры расчета сопротивления цепей тут.

  • Просмотров: 43551
  • простых параллельных схем | Последовательные и параллельные схемы

    На этой странице мы изложим три принципа, которые вы должны понимать в отношении параллельных цепей:

    1. Напряжение: Напряжение одинаково на всех компонентах в параллельной цепи.
    2. Ток: Полный ток цепи равен сумме токов отдельных ответвлений.
    3. Сопротивление: Отдельные сопротивления уменьшают , чтобы равняться меньшему общему сопротивлению, вместо прибавляют , чтобы получить общее.

    Давайте взглянем на несколько примеров параллельных цепей, демонстрирующих эти принципы.

    Начнем с параллельной схемы, состоящей из трех резисторов и одной батареи:

    Напряжение в параллельных цепях

    Первый принцип для понимания параллельных цепей заключается в том, что напряжение одинаково на всех компонентах в цепи . Это связано с тем, что в параллельной цепи есть только два набора электрически общих точек, и напряжение, измеренное между наборами общих точек, всегда должно быть одинаковым в любой момент времени.

    Следовательно, в приведенной выше схеме напряжение на R 1 равно напряжению на R 2 , которое равно напряжению на R 3 , которое равно напряжению на батарее.

    Это равенство напряжений можно представить в другой таблице для наших начальных значений:

    Применение закона Ома для простых параллельных цепей

    Как и в случае с последовательными цепями, применимо то же предостережение для закона Ома: значения напряжения, тока и сопротивления должны быть в одном контексте, чтобы вычисления работали правильно.

    Однако в приведенной выше примерной схеме мы можем немедленно применить закон Ома к каждому резистору, чтобы найти его ток, потому что мы знаем напряжение на каждом резисторе (9 вольт) и сопротивление каждого резистора:

    На данный момент мы все еще не знаем, каков полный ток или полное сопротивление для этой параллельной цепи, поэтому мы не можем применить закон Ома к крайнему правому столбцу («Всего»). Однако, если мы внимательно подумаем о том, что происходит, должно стать очевидным, что общий ток должен равняться сумме всех токов отдельных резисторов («ответвлений»):

    По мере того, как полный ток выходит из положительной (+) клеммы аккумулятора в точке 1 и проходит по цепи, часть потока разделяется в точке 2 и проходит через R 1 , еще часть разделяется в точке 3, чтобы пройти через R 2 , а оставшаяся часть идет через R 3 .Подобно реке, разветвляющейся на несколько более мелких ручьев, общий расход всех потоков должен равняться расходу всей реки.

    То же самое происходит, когда токи через R 1 , R 2 и R 3 соединяются, чтобы течь обратно к отрицательной клемме батареи (-) в направлении точки 8: поток тока из точки 7 до точки 8 должно равняться сумме токов (ответвлений) через R 1 , R 2 и R 3 .

    Это второй принцип параллельных цепей: полный ток цепи равен сумме токов отдельных ветвей .

    Используя этот принцип, мы можем заполнить место ИТ на нашем столе суммой I R1 , I R2 и I R3 :

    Как рассчитать полное сопротивление в параллельных цепях

    Наконец, применив закон Ома к крайнему правому столбцу («Всего»), мы можем вычислить полное сопротивление цепи:

    Уравнение сопротивления в параллельных цепях

    Обратите внимание на кое-что очень важное. Общее сопротивление цепи составляет всего 625 Ом: на меньше , чем у любого из отдельных резисторов.В последовательной цепи, где полное сопротивление было суммой отдельных сопротивлений, общее сопротивление должно было быть на больше , чем у любого из резисторов по отдельности.

    Здесь, в параллельной цепи, наоборот: мы говорим, что отдельных сопротивлений уменьшают , а не прибавляют , чтобы получилось .

    Этот принцип завершает нашу триаду «правил» для параллельных цепей, точно так же, как было обнаружено, что у последовательных цепей есть три правила для напряжения, тока и сопротивления.

    Математически соотношение между общим сопротивлением и отдельными сопротивлениями в параллельной цепи выглядит следующим образом:

    Как изменить схемы нумерации параллельных цепей для SPICE

    Та же основная форма уравнения работает для любого числа резисторов, соединенных вместе параллельно, просто добавьте столько членов 1 / R к знаменателю дроби, сколько необходимо для размещения всех параллельных резисторов в цепи.

    Как и в случае с последовательной схемой, мы можем использовать компьютерный анализ для перепроверки наших расчетов.Во-первых, конечно, мы должны описать нашу примерную схему компьютеру в понятных ему терминах. Я начну с рисования схемы:

    И снова мы обнаруживаем, что исходная схема нумерации, используемая для идентификации точек в цепи, должна быть изменена в интересах SPICE.

    В SPICE все электрически общие точки должны иметь одинаковые номера узлов. Вот как SPICE узнает, что с чем связано и как.

    В простой параллельной схеме все точки электрически являются общими в одном из двух наборов точек.В нашей примерной схеме провод, соединяющий верхние части всех компонентов, будет иметь один номер узла, а провод, соединяющий низ компонентов, будет иметь другой номер.

    Оставаясь верным соглашению о включении нуля в качестве номера узла, я выбираю числа 0 и 1:

    Пример, подобный этому, делает обоснование номеров узлов в SPICE довольно понятным. Поскольку все компоненты имеют общие наборы чисел, компьютер «знает», что все они подключены параллельно друг другу.

    Чтобы отобразить токи ответвлений в SPICE, нам нужно вставить источники нулевого напряжения последовательно (последовательно) с каждым резистором, а затем привязать наши измерения тока к этим источникам.

    По какой-то причине создатели программы SPICE сделали так, чтобы ток мог быть рассчитан только с по от источника напряжения. Это несколько раздражающее требование программы моделирования SPICE. После добавления каждого из этих «фиктивных» источников напряжения необходимо создать несколько новых номеров узлов, чтобы подключить их к соответствующим резисторам ответвления:

    Как проверить результаты компьютерного анализа

    Все фиктивные источники напряжения настроены на 0 В, чтобы не влиять на работу схемы.

    Файл описания схемы или список цепей выглядит так:

    Параллельная схема
     v1 1 0
     r1 2 0 10к
     r2 3 0 2k
     r3 4 0 1k
     vr1 1 2 постоянного тока 0
     vr2 1 3 постоянного тока 0
     vr3 1 4 постоянного тока 0
     .dc v1 9 9 1
     .print dc v (2,0) v (3,0) v (4,0)
     .print dc i (vr1) i (vr2) i (vr3)
     .конец
     

    Запустив компьютерный анализ, мы получили следующие результаты (я снабдил распечатку описательными этикетками):

    версия 1 v (2) v (3) v (4)
    9.000E + 00 9.000E + 00 9.000E + 00 9.000E + 00
    аккумулятор Напряжение R1 R2 напряжение Напряжение R3

    Напряжение

    версия 1 я (vr1) я (vr2) я (vr3)
    9.000E + 00 9.000E-04 4.500E-03 9.000E-03
    аккумулятор R1 ток R2 ток R3 ток

    Напряжение

    Эти значения действительно совпадают со значениями, вычисленными ранее по закону Ома: 0.9 мА для I R1 , 4,5 мА для I R2 и 9 мА для I R3 . При параллельном подключении, естественно, все резисторы имеют одинаковое падение напряжения на них (9 вольт, как у батареи).

    Три правила параллельных цепей

    Таким образом, параллельная цепь определяется как цепь, в которой все компоненты подключены между одним и тем же набором электрически общих точек. Другими словами, все компоненты подключены друг к другу через клеммы.Из этого определения следуют три правила параллельных цепей:

    • Все компоненты имеют одинаковое напряжение.
    • Сопротивления уменьшаются до меньшего общего сопротивления.
    • Токи ответвления в сумме равняются большему общему току.

    Как и в случае с последовательными цепями, все эти правила находят корень в определении параллельной цепи. Если вы полностью понимаете это определение, то правила – не что иное, как сноски к определению.

    ОБЗОР:

    • Компоненты в параллельной цепи имеют одинаковое напряжение: E Всего = E 1 = E 2 =. . . E n
    • Общее сопротивление в параллельной цепи на меньше , чем любое из отдельных сопротивлений: R Всего = 1 / (1 / R 1 + 1 / R 2 +… 1 / R n )
    • Общий ток в параллельной цепи равен сумме токов отдельных ответвлений: I Всего = I 1 + I 2 +.. . Я н. .

    СВЯЗАННЫЕ РАБОЧИЕ ЛИСТЫ:

    Формулы и калькулятор »Электроника

    Формулы, расчеты и калькулятор для определения общего сопротивления резисторов, установленных последовательно и параллельно.


    Resistance Tutorial:
    Что такое сопротивление Закон Ома Омические и неомические проводники Сопротивление лампы накаливания Удельное сопротивление Таблица удельного сопротивления для распространенных материалов Температурный коэффициент сопротивления Электрическая проводимость Последовательные и параллельные резисторы Таблица параллельных резисторов


    Резисторы могут быть размещены во многих конфигурациях в электрической или электронной схеме – иногда последовательно, иногда параллельно.

    Когда они размещаются в этих конфигурациях, важно иметь возможность рассчитать общее сопротивление. Этого можно довольно легко достичь, если использовать правильные формулы – есть простые формулы как для последовательных, так и для параллельных резисторов.

    При проектировании электронной схемы или по какой-либо другой причине возможность вычисления сопротивления комбинации резисторов может быть очень полезной.

    В электронных схемах комбинации резисторов могут быть сведены к последовательным элементам и параллельным элементам, хотя при использовании других электронных компонентов комбинации могут быть более сложными.Однако во многих случаях расчет значений последовательного и параллельного сопротивления имеет большое значение.


    Посмотрите наше видео о расчетах последовательных и параллельных резисторов

    Последовательные резисторы

    Самая простая конфигурация электронной схемы – это резисторы, включенные последовательно. Это может произойти, если несколько этих электронных компонентов соединены последовательно, или необходимо добавить сопротивление кабеля к сопротивлению резистора и т. Д.

    Если резисторы соединены последовательно, то общее сопротивление является просто суммой отдельных резисторов.

    Последовательные резисторы

    Величину резисторов или сопротивлений, включенных последовательно, можно математически выразить следующим образом:

    Пример расчета последовательных резисторов:
    В качестве примера, если три резистора, имеющие номиналы 1 кОм, 2 кОм и 3 кОм, соединены последовательно, то общее сопротивление составит 1 + 2 + 3 кОм = 6 кОм.

    В реальных жизненных ситуациях и аспектах проектирования электрических и электронных схем будет много областей, где есть электронные компоненты, такие как резисторы или другие элементы, вносящие сопротивление, где количество последовательно соединенных сопротивлений, которые необходимо суммировать.

    Резисторы параллельно

    Есть также много случаев, когда электронные компоненты, такие как резисторы, а также другие элементы, вызывающие сопротивление, появляются в электрической или электронной цепи параллельно.

    Если резисторы размещены параллельно, они разделяют ток, и ситуацию немного сложнее вычислить, но все же довольно легко.

    1Rtotal = 1R1 + 1R2 + 1R3 + ……

    Пример расчета сопротивления резисторов, включенных параллельно:
    В качестве примера, если есть три резистора, подключенных параллельно со значениями 1 кОм, 2 кОм и Омега и 3 кОм, то можно вычислить общее значение комбинации:

    1 / R Итого = 1/1000 + 1/2000 + 1/3000

    1 / R Итого = 1/1000 + 1/2000 + 1/3000

    1 / R Итого = 6/6000 + 3/6000 + 2/6000

    1 / R Итого = 11/6000

    R Всего = 6000/11 Ом или 545 Ом

    Корпус только двух резисторов, включенных параллельно

    Во многих конструкциях электронных схем наиболее распространенный экземпляр параллельных резисторов состоит только из двух электронных компонентов.

    Часто бывает так, что один резистор подключается параллельно другому. Или другой случай может быть, когда резистор помещается на клеммы для цепи или сети, которая имеет определенное сопротивление. В этом случае необходимо только рассчитать общее сопротивление для двух параллельно включенных резисторов.

    Если необходимо рассчитать общее значение для двух параллельных резисторов, уравнением можно манипулировать и значительно упростить его, как показано ниже:

    Эта формула значительно упрощает вычисление номинала двух параллельных резисторов, так как требует только одного умножения, одного сложения и одного деления.Часто это можно сделать мысленно или на клочке бумаги. В качестве альтернативы можно использовать наш простой калькулятор для двух параллельно включенных резисторов, приведенный ниже.

    Калькулятор для двух резисторов, включенных параллельно

    Этот калькулятор параллельного сопротивления обеспечивает простой метод расчета общего сопротивления для двух резисторов, соединенных параллельно.

    Хотя параллельный расчет номиналов резисторов для двух резисторов упрощается до простой формулы, иногда гораздо проще и быстрее использовать калькулятор.

    Чтобы использовать калькулятор параллельных резисторов, просто введите значения параллельных резисторов в Ом, Ом или кОм и т. Д. В два поля ввода, но обратите внимание, что все значения должны быть в одних и тех же единицах, то есть оба в Ом кОм МОм и т. Д. Затем вычислитель параллельных резисторов предоставит общее сопротивление двух резисторов в тех же единицах, что и вход.

    Введите два значения для резисторов, R1 и R2, в поля, представленные в калькуляторе ниже, нажмите «Рассчитать», и будет предоставлено общее сопротивление.


    Калькулятор параллельного сопротивления

    Калькулятор параллельных резисторов обеспечивает простой способ рассчитать сопротивление двух резисторов, включенных параллельно, экономя записывать все и прибегая к ручке и бумаге или калькулятору в той или иной форме.

    Знание того, как рассчитывать значения резисторов, включенных последовательно и параллельно, является ключом к пониманию того, как работают электрические и электронные схемы. Эти концепции используются как вторая натура в проектировании электрических и электронных схем.

    Дополнительные концепции и руководства по основам электроники:
    Voltage Текущий Власть Сопротивление Емкость Индуктивность Трансформеры Децибел, дБ Законы Кирхгофа Q, добротность Радиочастотный шум
    Вернуться в меню «Основные понятия электроники». . .

    Как рассчитать резисторы, включенные последовательно и параллельно – Kitronik Ltd

    Резисторы

    серии

    Когда резисторы подключаются друг за другом, это называется последовательным соединением.Это показано ниже. Чтобы рассчитать общее общее сопротивление ряда резисторов, подключенных таким образом, вы складываете отдельные сопротивления. Это делается по следующей формуле: Rtotal = R1 + R2 + R3 и так далее. Пример: чтобы рассчитать полное сопротивление для этих трех последовательно соединенных резисторов.
    Rtotal = R1 + R2 + R3 = 100 + 82 + 1 Ом = 183 Ом

    Задача 1:

    Рассчитайте общее сопротивление следующего последовательно включенного резистора.
    R Итого = _______________
    = _______________
    R Итого = _______________
    = _______________
    R Итого = _______________
    = _______________

    Параллельные резисторы

    Когда резисторы подключаются друг к другу (бок о бок), это называется параллельным подключением.Это показано ниже.

    Два параллельных резистора

    Для расчета общего полного сопротивления a двух резисторов, подключенных таким образом, вы можете использовать следующую формулу:
    Пример: чтобы рассчитать полное сопротивление для этих двух резисторов, включенных параллельно.

    Задача 2:

    Рассчитайте полное сопротивление следующего резистора, включенного параллельно.

    Три или более резистора, включенных параллельно

    Для расчета общего общего сопротивления ряда из трех или более резисторов, подключенных таким образом, вы можете использовать следующую формулу: Пример: Чтобы рассчитать общее сопротивление для этих трех резисторов, подключенных параллельно

    Задача 3:

    Рассчитайте полное сопротивление следующего резистора, включенного параллельно.

    ответов

    Задача 1

    1 = 1492 Ом 2 = 2242 Ом 3 = 4847 Ом

    Задача 2

    1 = 5 Ом 2 = 9,57 Ом 3 = 248,12 Ом

    Задача 3

    1 = 5,95 Ом 2 = 23,76 Ом Загрузите pdf-версию этой страницы здесь. Узнать больше об авторе подробнее »

    © Kitronik Ltd – Вы можете распечатать эту страницу и ссылку на нее, но не должны копировать страницу или ее часть без предварительного письменного согласия Kitronik.

    6.2 Последовательные и параллельные резисторы – Введение в электричество, магнетизм и схемы

    ЦЕЛИ ОБУЧЕНИЯ

    К концу раздела вы сможете:
    • Определите термин эквивалентное сопротивление
    • Рассчитайте эквивалентное сопротивление резисторов, включенных последовательно
    • Рассчитайте эквивалентное сопротивление резисторов, включенных параллельно

    В статье «Ток и сопротивление» мы описали термин «сопротивление» и объяснили основную конструкцию резистора.По сути, резистор ограничивает поток заряда в цепи и представляет собой омическое устройство, где. В большинстве схем имеется более одного резистора. Если несколько резисторов соединены вместе и подключены к батарее, ток, подаваемый батареей, зависит от эквивалентного сопротивления цепи.

    Эквивалентное сопротивление комбинации резисторов зависит как от их индивидуальных значений, так и от способа их подключения. Самыми простыми комбинациями резисторов являются последовательное и параллельное соединение (рисунок 6.2.1). В схеме серии выходной ток первого резистора течет на вход второго резистора; следовательно, ток в каждом резисторе одинаков. В параллельной схеме все выводы резистора на одной стороне резисторов соединены вместе, а все выводы на другой стороне соединены вместе. В случае параллельной конфигурации каждый резистор имеет одинаковое падение потенциала на нем, и токи через каждый резистор могут быть разными, в зависимости от резистора.Сумма отдельных токов равна току, протекающему по параллельным соединениям.

    (рисунок 6.2.1)

    Рисунок 6.2.1 (a) При последовательном соединении резисторов ток одинаков на каждом резисторе. (b) При параллельном соединении резисторов напряжение на каждом резисторе одинаковое.

    Резисторы серии

    Считается, что резисторы

    включены последовательно, если ток течет через резисторы последовательно. Рассмотрим рисунок 6.2.2, на котором показаны три последовательно включенных резистора с приложенным напряжением, равным.Поскольку заряды проходят только по одному пути, ток через каждый резистор одинаков. Эквивалентное сопротивление набора резисторов при последовательном соединении равно алгебраической сумме отдельных сопротивлений.

    (рисунок 6.2.2)

    Рисунок 6.2.2 (a) Три резистора, подключенные последовательно к источнику напряжения. (b) Исходная схема сокращается до эквивалентного сопротивления и источника напряжения.

    На рисунке 6.2.2 ток, идущий от источника напряжения, протекает через каждый резистор, поэтому ток через каждый резистор одинаков.Ток в цепи зависит от напряжения, подаваемого источником напряжения, и сопротивления резисторов. Для каждого резистора происходит падение потенциала, равное потере электрической потенциальной энергии при прохождении тока через каждый резистор. Согласно закону Ома падение потенциала на резисторе при протекании через него тока рассчитывается по формуле, где – ток в амперах (), а – сопротивление в омах (). Поскольку энергия сохраняется, а напряжение равно потенциальной энергии на заряд, сумма напряжения, приложенного к цепи источником, и падения потенциала на отдельных резисторах вокруг контура должны быть равны нулю:

    Это уравнение часто называют законом петли Кирхгофа, который мы рассмотрим более подробно позже в этой главе.На рисунке 6.2.2 сумма падения потенциала каждого резистора и напряжения, подаваемого источником напряжения, должна равняться нулю:

    Поскольку ток через каждый компонент одинаков, равенство можно упростить до эквивалентного сопротивления, которое представляет собой просто сумму сопротивлений отдельных резисторов.

    Любое количество резисторов может быть подключено последовательно. Если резисторы соединены последовательно, эквивалентное сопротивление равно

    .

    (6.2.1)

    Одним из результатов включения компонентов в последовательную цепь является то, что если что-то происходит с одним компонентом, это влияет на все остальные компоненты. Например, если несколько ламп подключены последовательно и одна лампа перегорела, все остальные лампы погаснут.

    ПРИМЕР 6.2.1


    Эквивалентное сопротивление, ток и мощность
    в последовательной цепи

    Батарея с напряжением на клеммах подключена к цепи, состоящей из четырех и одного последовательно соединенных резисторов (рисунок 6.2.3). Предположим, что батарея имеет незначительное внутреннее сопротивление. (а) Рассчитайте эквивалентное сопротивление цепи. (b) Рассчитайте ток через каждый резистор. (c) Рассчитайте падение потенциала на каждом резисторе. (d) Определите общую мощность, рассеиваемую резисторами, и мощность, потребляемую батареей.

    (рисунок 6.2.3)

    Рисунок 6.2.3 Простая последовательная схема с пятью резисторами.
    Стратегия

    В последовательной цепи эквивалентное сопротивление представляет собой алгебраическую сумму сопротивлений.Ток в цепи можно найти из закона Ома и равен напряжению, деленному на эквивалентное сопротивление. Падение потенциала на каждом резисторе можно найти с помощью закона Ома. Мощность, рассеиваемая каждым резистором, может быть найдена с помощью, а общая мощность, рассеиваемая резисторами, равна сумме мощности, рассеиваемой каждым резистором. Мощность, обеспечиваемую аккумулятором, можно найти с помощью.

    Решение

    а. Эквивалентное сопротивление – это алгебраическая сумма сопротивлений:

    г.Ток в цепи одинаков для каждого резистора в последовательной цепи и равен приложенному напряжению, деленному на эквивалентное сопротивление:

    г. Падение потенциала на каждом резисторе можно найти с помощью закона Ома:

    Обратите внимание, что сумма падений потенциала на каждом резисторе равна напряжению, подаваемому батареей.

    г. Мощность, рассеиваемая резистором, равна, а мощность, отдаваемая аккумулятором, равна:

    Значение

    Есть несколько причин, по которым мы использовали бы несколько резисторов вместо одного резистора с сопротивлением, равным эквивалентному сопротивлению цепи.Возможно, резистора необходимого размера нет в наличии, или нам нужно отводить выделяемое тепло, или мы хотим минимизировать стоимость резисторов. Каждый резистор может стоить от нескольких центов до нескольких долларов, но при умножении на тысячи единиц экономия затрат может быть значительной.

    ПРОВЕРЬТЕ ПОНИМАНИЕ 6.2

    Некоторые гирлянды миниатюрных праздничных огней закорачиваются при перегорании лампочки. Устройство, вызывающее короткое замыкание, называется шунтом, который позволяет току течь по разомкнутой цепи.«Короткое замыкание» похоже на протягивание куска проволоки через компонент. Луковицы обычно сгруппированы в серию по девять луковиц. Если перегорает слишком много лампочек, в конечном итоге открываются шунты. Что вызывает это?

    Кратко обозначим основные характеристики последовательно соединенных резисторов:

    Сопротивления серии
    1. суммируются, чтобы получить эквивалентное сопротивление:

    2. Одинаковый ток протекает последовательно через каждый резистор.
    3. Отдельные последовательно включенные резисторы не получают полное напряжение источника, а делят его.Общее падение потенциала на последовательной конфигурации резисторов равно сумме падений потенциала на каждом резисторе.

    Параллельные резисторы

    На рисунке 6.2.4 показаны резисторы, включенные параллельно, подключенные к источнику напряжения. Резисторы включены параллельно, когда один конец всех резисторов соединен непрерывным проводом с незначительным сопротивлением, а другой конец всех резисторов также соединен друг с другом непрерывным проводом с незначительным сопротивлением.Падение потенциала на каждом резисторе одинаковое. Ток через каждый резистор можно найти с помощью закона Ома, где напряжение на каждом резисторе постоянно. Например, автомобильные фары, радио и другие системы подключены параллельно, так что каждая подсистема использует полное напряжение источника и может работать полностью независимо. То же самое и с электропроводкой в ​​вашем доме или любом здании.

    (рисунок 6.2.4)

    Рисунок 6.2.4 (a) Два резистора, подключенных параллельно источнику напряжения.(b) Исходная схема сокращается до эквивалентного сопротивления и источника напряжения.

    Ток, протекающий от источника напряжения на рисунке 6.2.4, зависит от напряжения, подаваемого источником напряжения, и эквивалентного сопротивления цепи. В этом случае ток течет от источника напряжения и попадает в переход или узел, где цепь разделяется, протекая через резисторы и. По мере того, как заряды проходят от батареи, некоторые проходят через резистор, а некоторые – через резистор. Сумма токов, протекающих в переходе, должна быть равна сумме токов, текущих из перехода:

    Это уравнение называется правилом соединения Кирхгофа и будет подробно обсуждено в следующем разделе.На рисунке 6.2.4 показано правило соединения. В этой схеме есть две петли, которые приводят к уравнениям и Обратите внимание, что напряжение на резисторах, включенных параллельно, одинаковое (), а ток является аддитивным:

    Если обобщить на любое количество резисторов, эквивалентное сопротивление параллельного соединения связано с отдельными сопротивлениями соотношением

    .

    (6.2.2)

    Это соотношение приводит к эквивалентному сопротивлению, которое меньше наименьшего из отдельных сопротивлений.Когда резисторы подключены параллельно, от источника течет больше тока, чем протекает для любого из них по отдельности, поэтому общее сопротивление ниже.

    ПРИМЕР 6.2.2


    Анализ параллельной цепи

    Три резистора, и соединены параллельно. Параллельное соединение подключается к источнику напряжения. а) Какое эквивалентное сопротивление? (b) Найдите ток, подаваемый источником в параллельную цепь. (c) Рассчитайте токи в каждом резисторе и покажите, что в сумме они равны выходному току источника.(d) Рассчитайте мощность, рассеиваемую каждым резистором. (e) Найдите выходную мощность источника и покажите, что она равна общей мощности, рассеиваемой резисторами.

    Стратегия

    (a) Общее сопротивление для параллельной комбинации резисторов определяется с помощью.
    (Обратите внимание, что в этих вычислениях каждый промежуточный ответ отображается с дополнительной цифрой.)

    (b) Ток, подаваемый источником, можно найти из закона Ома, заменив полное сопротивление.

    (c) Отдельные токи легко вычислить по закону Ома, поскольку каждый резистор получает полное напряжение.Полный ток – это сумма отдельных токов:.

    (d) Мощность, рассеиваемую каждым резистором, можно найти с помощью любого из уравнений, связывающих мощность с током, напряжением и сопротивлением, поскольку все три известны. Давайте использовать, так как каждый резистор получает полное напряжение.

    (e) Общую мощность также можно рассчитать несколькими способами, используйте.

    Решение

    а. Общее сопротивление для параллельной комбинации резисторов находится с помощью уравнения 6.2.2.Ввод известных значений дает

    Общее сопротивление с правильным количеством значащих цифр составляет. Как и предполагалось, меньше минимального индивидуального сопротивления.

    г. Полный ток можно найти из закона Ома, заменив полное сопротивление. Это дает

    Ток для каждого устройства намного больше, чем для тех же устройств, подключенных последовательно (см. Предыдущий пример). Схема с параллельным соединением имеет меньшее общее сопротивление, чем резисторы, включенные последовательно.

    г. Отдельные токи легко вычислить по закону Ома, поскольку на каждый резистор подается полное напряжение. Таким образом,

    Аналогично

    и

    Общий ток складывается из отдельных токов:

    г. Мощность, рассеиваемую каждым резистором, можно найти с помощью любого из уравнений, связывающих мощность с током, напряжением и сопротивлением, поскольку все три известны.Давайте использовать, так как каждый резистор получает полное напряжение. Таким образом,

    Аналогично

    и

    e. Суммарную мощность также можно рассчитать несколькими способами. Выбор и ввод общей текущей доходности

    Значение

    Общая мощность, рассеиваемая резисторами, также составляет:

    Обратите внимание, что общая мощность, рассеиваемая резисторами, равна мощности, подаваемой источником.

    ПРОВЕРЬТЕ ПОНИМАНИЕ 6.3


    Рассмотрим одну и ту же разность потенциалов, приложенную к одним и тем же трем последовательно включенным резисторам. Будет ли эквивалентное сопротивление последовательной цепи больше, меньше или равно трем резисторам, включенным параллельно? Будет ли ток в последовательной цепи выше, ниже или равен току, обеспечиваемому тем же напряжением, приложенным к параллельной цепи? Как мощность, рассеиваемая последовательно подключенными резисторами, будет сравниваться с мощностью, рассеиваемой параллельно резисторами?

    ПРОВЕРЬТЕ ПОНИМАНИЕ 6.4


    Как бы вы использовали реку и два водопада, чтобы смоделировать параллельную конфигурацию двух резисторов? Как разрушается эта аналогия?

    Суммируем основные характеристики резисторов параллельно:

    1. Эквивалентное сопротивление находится из

      и меньше любого отдельного сопротивления в комбинации.

    2. Падение потенциала на каждом параллельном резисторе одинаковое.
    3. Параллельные резисторы не получают суммарный ток каждый; они делят это.Ток, поступающий в параллельную комбинацию резисторов, равен сумме токов, протекающих через каждый резистор, включенный параллельно.

    В этой главе мы представили эквивалентное сопротивление резисторов, соединенных последовательно, и резисторов, соединенных параллельно. Вы можете вспомнить, что в разделе «Емкость» мы ввели эквивалентную емкость конденсаторов, соединенных последовательно и параллельно. Цепи часто содержат как конденсаторы, так и резисторы. В таблице 6.2.1 приведены уравнения, используемые для эквивалентного сопротивления и эквивалентной емкости для последовательных и параллельных соединений.

    (таблица 6.2.1)

    Комбинация серии Параллельная комбинация
    Эквивалентная емкость
    Эквивалентное сопротивление

    Таблица 10.1 Сводка по эквивалентному сопротивлению и емкости в последовательной и параллельной комбинациях

    Сочетания последовательного и параллельного

    Более сложные соединения резисторов часто представляют собой просто комбинации последовательного и параллельного соединения.Такие комбинации обычны, особенно если учесть сопротивление проводов. В этом случае сопротивление провода включено последовательно с другими сопротивлениями, включенными параллельно.

    Комбинации последовательного и параллельного соединения можно уменьшить до одного эквивалентного сопротивления, используя метод, показанный на Рисунке 6.2.5. Различные части могут быть идентифицированы как последовательные или параллельные соединения, уменьшенные до их эквивалентных сопротивлений, а затем уменьшенные до тех пор, пока не останется единственное эквивалентное сопротивление. Процесс занимает больше времени, чем труден.Здесь мы отмечаем эквивалентное сопротивление как.

    (рисунок 6.2.5)

    Обратите внимание, что резисторы и включены последовательно. Их можно объединить в одно эквивалентное сопротивление. Один из методов отслеживания процесса – включить резисторы в качестве индексов. Здесь эквивалентное сопротивление и равно

    .

    Теперь схема сокращается до трех резисторов, показанных на Рисунке 6.2.5 (c). Перерисовывая, мы теперь видим, что резисторы и составляют параллельную цепь.Эти два резистора можно уменьшить до эквивалентного сопротивления:

    .

    Этот шаг процесса сокращает схему до двух резисторов, показанных на Рисунке 6.2.5 (d). Здесь схема сводится к двум резисторам, которые в данном случае включены последовательно. Эти два резистора можно уменьшить до эквивалентного сопротивления, которое является эквивалентным сопротивлением цепи:

    Основная цель этого анализа схемы достигнута, и теперь схема сводится к одному резистору и одному источнику напряжения.

    Теперь мы можем проанализировать схему. Ток, обеспечиваемый источником напряжения, равен. Этот ток проходит через резистор и обозначен как. Падение потенциала можно найти с помощью закона Ома:

    Глядя на рис. 6.2.5 (c), остается отбросить параллельную комбинацию и. Проходной ток можно найти с помощью закона Ома:

    Резисторы и включены последовательно, поэтому токи и равны

    .

    Используя закон Ома, мы можем найти падение потенциала на двух последних резисторах.Потенциальные падения равны и. Окончательный анализ – это посмотреть на мощность, подаваемую источником напряжения, и мощность, рассеиваемую резисторами. Мощность, рассеиваемая резисторами

    Полная энергия постоянна в любом процессе. Следовательно, мощность, подаваемая источником напряжения, равна. Анализ мощности, подаваемой в схему, и мощности, рассеиваемой резисторами, является хорошей проверкой достоверности анализа; они должны быть равны.

    ПРОВЕРЬТЕ ПОНИМАНИЕ 6.5


    Рассмотрите электрические цепи в вашем доме. Приведите по крайней мере два примера схем, которые должны использовать комбинацию последовательных и параллельных схем для эффективной работы.

    Практическое применение

    Одним из следствий этого последнего примера является то, что сопротивление в проводах снижает ток и мощность, подаваемую на резистор. Если сопротивление провода относительно велико, как в изношенном (или очень длинном) удлинителе, то эти потери могут быть значительными. Если протекает большой ток, провал в проводах также может быть значительным и проявляться в виде тепла, выделяемого в шнуре.

    Например, когда вы роетесь в холодильнике и включается мотор, свет холодильника на мгновение гаснет. Точно так же вы можете увидеть тусклый свет в салоне, когда вы запускаете двигатель вашего автомобиля (хотя это может быть связано с сопротивлением внутри самой батареи).

    Что происходит в этих сильноточных ситуациях, показано на Рисунке 6.2.7. Устройство, представленное значком, имеет очень низкое сопротивление, поэтому при его включении протекает большой ток.Этот увеличенный ток вызывает большее падение в проводах, представленных значком, уменьшая напряжение на лампочке (которая есть), которое затем заметно гаснет.

    (рисунок 6.2.7)

    Рисунок 6.2.7 Почему свет тускнеет, когда включен большой прибор? Ответ заключается в том, что большой ток, потребляемый двигателем прибора, вызывает значительное падение напряжения в проводах и снижает напряжение на свету.

    Стратегия решения проблем: последовательные и параллельные резисторы


    1. Нарисуйте четкую принципиальную схему, пометив все резисторы и источники напряжения.Этот шаг включает список известных значений проблемы, поскольку они отмечены на вашей принципиальной схеме.
    2. Определите, что именно необходимо определить в проблеме (определите неизвестные). Письменный список полезен.
    3. Определите, включены ли резисторы последовательно, параллельно или в комбинации последовательно и параллельно. Изучите принципиальную схему, чтобы сделать эту оценку. Резисторы включены последовательно, если через них должен последовательно проходить один и тот же ток.
    4. Используйте соответствующий список основных функций для последовательных или параллельных подключений, чтобы найти неизвестные.Есть один список для серий, а другой – для параллелей.
    5. Проверьте, являются ли ответы разумными и последовательными.

    ПРИМЕР 6.2.4


    Объединение последовательных и параллельных цепей

    Два резистора, соединенных последовательно, подключены к двум резисторам, включенным параллельно. Последовательно-параллельная комбинация подключается к батарее. Каждый резистор имеет сопротивление. Провода, соединяющие резисторы и аккумулятор, имеют незначительное сопротивление.Ток проходит через резистор. Какое напряжение подается от источника напряжения?

    Стратегия

    Используйте шаги предыдущей стратегии решения проблем, чтобы найти решение для этого примера.

    Решение
    1. Нарисуйте четкую принципиальную схему (рисунок 6.2.8).

      (рисунок 6.2.8)

      Рисунок 6.2.8 Чтобы найти неизвестное напряжение, мы должны сначала найти эквивалентное сопротивление цепи.
    2. Неизвестно напряжение аккумулятора.Чтобы определить напряжение, подаваемое батареей, необходимо найти эквивалентное сопротивление.
    3. В этой схеме мы уже знаем, что резисторы и включены последовательно, а резисторы и включены параллельно. Эквивалентное сопротивление параллельной конфигурации резисторов и последовательно с последовательной конфигурацией резисторов и.
    4. Напряжение, подаваемое батареей, можно найти, умножив ток от батареи на эквивалентное сопротивление цепи.Ток от аккумулятора равен току через него и равен. Нам нужно найти эквивалентное сопротивление, уменьшив схему. Чтобы уменьшить схему, сначала рассмотрите два резистора, включенных параллельно. Эквивалентное сопротивление составляет. Эта параллельная комбинация включена последовательно с двумя другими резисторами, поэтому эквивалентное сопротивление цепи равно. Таким образом, напряжение, подаваемое батареей, составляет.
    5. Один из способов проверить соответствие ваших результатов – это рассчитать мощность, подаваемую батареей, и мощность, рассеиваемую резисторами.Мощность, подаваемая аккумулятором, составляет

      Поскольку они включены последовательно, сквозной ток равен сквозному току. Т.к. ток через каждый будет. Мощность, рассеиваемая резисторами, равна сумме мощности, рассеиваемой каждым резистором:


      Поскольку мощность, рассеиваемая резисторами, равна мощности, подаваемой батареей, наше решение кажется последовательным.

    Значение

    Если проблема имеет комбинацию последовательного и параллельного соединения, как в этом примере, ее можно уменьшить поэтапно, используя предыдущую стратегию решения проблемы и рассматривая отдельные группы последовательных или параллельных соединений.При поиске параллельного подключения необходимо соблюдать осторожность. Кроме того, единицы и числовые результаты должны быть разумными. Эквивалентное последовательное сопротивление должно быть больше, а эквивалентное параллельное сопротивление, например, должно быть меньше. Мощность должна быть больше для одних и тех же устройств, подключенных параллельно, по сравнению с последовательными и т. Д.

    Кандела Цитаты

    Лицензионный контент CC, особая атрибуция

    • Загрузите бесплатно по адресу http: // cnx.org/contents/[email protected]. Получено с : http://cnx.org/contents/[email protected]. Лицензия : CC BY: Attribution

    резисторов последовательно и параллельно

    Резисторы серии

    Общее сопротивление в цепи с последовательно включенными резисторами равно сумме отдельных сопротивлений.

    Цели обучения

    Рассчитайте общее сопротивление в цепи с последовательно включенными резисторами

    Основные выводы

    Ключевые моменты
    • Одинаковый ток протекает последовательно через каждый резистор.
    • Отдельные последовательно включенные резисторы не получают полное напряжение источника, а делят его.
    • Общее сопротивление в последовательной цепи равно сумме отдельных сопротивлений: [латекс] \ text {RN} (\ text {series}) = \ text {R} _1 + \ text {R} _2 + \ text {R} _3 +… + \ text {R} _ \ text {N} [/ latex].
    Ключевые термины
    • серия : ряд элементов, которые следуют одно за другим или связаны друг за другом.
    • сопротивление : Противодействие прохождению электрического тока через этот элемент.

    Обзор

    Большинство схем имеет более одного компонента, называемого резистором, который ограничивает поток заряда в цепи. Мера этого предела для потока заряда называется сопротивлением. Самыми простыми комбинациями резисторов являются последовательное и параллельное соединение. Общее сопротивление комбинации резисторов зависит как от их индивидуальных значений, так и от способа их подключения.

    Цепи серии : Краткое введение в анализ последовательных и последовательных цепей, включая закон Кирхгофа по току (KCL) и закон Кирхгофа по напряжению (KVL).

    Резисторы серии

    Резисторы включены последовательно всякий раз, когда заряд или ток должны проходить через компоненты последовательно.

    Резисторы в серии : Эти четыре резистора подключены последовательно, потому что, если бы ток подавался на один конец, он бы протекал через каждый резистор последовательно до конца.

    показывает резисторы, последовательно подключенные к источнику напряжения. Общее сопротивление в цепи равно сумме отдельных сопротивлений, поскольку ток должен последовательно проходить через каждый резистор.

    Резисторы, подключенные последовательно. : Три резистора, подключенные последовательно к батарее (слева), и эквивалентное одиночное или последовательное сопротивление (справа).

    Использование закона Ома для расчета изменений напряжения в резисторах серии

    Согласно закону Ома падение напряжения V на резисторе при протекании через него тока рассчитывается по формуле V = IR, где I – ток в амперах (A), а R – сопротивление в омах (Ω). .

    Таким образом, падение напряжения на R 1 равно В 1 = IR 1 , на R 2 равно V 2 = IR 2 , а на R 3 равно V 3 = IR 3 .Сумма напряжений будет равна: V = V 1 + V 2 + V 3 , исходя из сохранения энергии и заряда. Если подставить значения отдельных напряжений, получим:

    [латекс] \ text {V} = \ text {IR} _1 + \ text {IR} _2 + \ text {IR} _3 [/ latex]

    или

    [латекс] \ text {V} = \ text {I} (\ text {R} _1 + \ text {R} _2 + \ text {R} _3) [/ латекс]

    Это означает, что полное сопротивление в серии равно сумме отдельных сопротивлений. Следовательно, для каждой цепи с Н количество резисторов, включенных последовательно:

    [латекс] \ text {RN} (\ text {series}) = \ text {R} _1 + \ text {R} _2 + \ text {R} _3 +… + \ text {R} _ \ text {N }.[/ латекс]

    Поскольку весь ток должен проходить через каждый резистор, он испытывает сопротивление каждого, и последовательно соединенные сопротивления просто складываются.

    Поскольку напряжение и сопротивление имеют обратную зависимость, отдельные последовательно включенные резисторы не получают полное напряжение источника, а делят его. Об этом свидетельствует пример, когда две лампочки соединены в последовательную цепь с аккумулятором. В простой схеме, состоящей из одной батареи 1,5 В и одной лампочки, падение напряжения на лампе будет равно 1.5V через него. Однако, если бы две лампочки были соединены последовательно с одной и той же батареей, на каждой из них было бы падение напряжения 1,5 В / 2 или 0,75 В. Это будет очевидно по яркости света: каждая из двух последовательно соединенных лампочек будет в два раза слабее, чем одиночная лампочка. Следовательно, резисторы, соединенные последовательно, потребляют такое же количество энергии, как и один резистор, но эта энергия распределяется между резисторами в зависимости от их сопротивлений.

    Параллельные резисторы

    Общее сопротивление в параллельной цепи равно сумме обратных сопротивлений каждого отдельного сопротивления.

    Цели обучения

    Рассчитайте общее сопротивление в цепи с резисторами, включенными параллельно

    Основные выводы

    Ключевые моменты
    • Общее сопротивление в параллельной цепи меньше наименьшего из отдельных сопротивлений.
    • Каждый резистор, включенный параллельно, имеет то же напряжение, что и приложенный к нему источник (напряжение в параллельной цепи постоянно).
    • Параллельные резисторы не получают суммарный ток каждый; они делят его (ток зависит от номинала каждого резистора и общего количества резисторов в цепи).
    Ключевые термины
    • сопротивление : Противодействие прохождению электрического тока через этот элемент.
    • параллельно : Расположение электрических компонентов, при котором ток течет по двум или более путям.

    Обзор

    Резисторы в цепи могут быть включены последовательно или параллельно. Общее сопротивление комбинации резисторов зависит как от их индивидуальных значений, так и от способа их подключения.

    Parallel Circuits : Краткий обзор анализа параллельных цепей с использованием таблиц VIRP для студентов-физиков средней школы.

    Параллельные резисторы

    Резисторы включены параллельно, когда каждый резистор подключен непосредственно к источнику напряжения путем соединения проводов с незначительным сопротивлением. Таким образом, к каждому резистору приложено полное напряжение источника.

    Параллельное соединение резисторов : Параллельное соединение резисторов.

    Каждый резистор потребляет такой же ток, как если бы он был единственным резистором, подключенным к источнику напряжения. Это верно для схем в доме или квартире. Каждая розетка, подключенная к устройству («резистор»), может работать независимо, и ток не должен проходить через каждое устройство последовательно.

    Закон и параллельные резисторы Ом

    На каждый резистор в цепи подается полное напряжение. Согласно закону Ома токи, протекающие через отдельные резисторы, равны [латекс] \ text {I} _1 = \ frac {\ text {V}} {\ text {R} _1} [/ latex], [latex] \ text {I} _2 = \ frac {\ text {V}} {\ text {R} _2} [/ latex] и [latex] \ text {I} _3 = \ frac {\ text {V}} {\ text {R} _3} [/ латекс].Сохранение заряда подразумевает, что полный ток является суммой этих токов:

    Параллельные резисторы : три резистора, подключенные параллельно батарее, и эквивалентное одиночное или параллельное сопротивление.

    [латекс] \ text {I} = \ text {I} _1 + \ text {I} _2 + \ text {I} _3. [/ Latex]

    Подстановка выражений для отдельных токов дает:

    [латекс] \ text {I} = \ frac {\ text {V}} {\ text {R} _1} + \ frac {\ text {V}} {\ text {R} _2} + \ frac {\ текст {V}} {\ text {R} _3} [/ latex]

    или

    [латекс] \ text {I} = \ text {V} (\ frac {1} {\ text {R} _1} + \ frac {1} {\ text {R} _2} + \ frac {1} { \ text {R} _3}) [/ latex]

    Это означает, что полное сопротивление в параллельной цепи равно сумме обратных сопротивлений каждого отдельного сопротивления.Следовательно, для каждой схемы с числом [latex] \ text {n} [/ latex] или резисторов, подключенных параллельно,

    [латекс] \ text {R} _ {\ text {n} \; (\ text {parallel})} = \ frac {1} {\ text {R} _1} + \ frac {1} {\ text { R} _2} + \ frac {1} {\ text {R} _3}… + \ frac {1} {\ text {R} _ \ text {n}}. [/ Latex]

    Это соотношение приводит к общему сопротивлению, которое меньше наименьшего из отдельных сопротивлений. Когда резисторы подключены параллельно, от источника течет больше тока, чем протекает для любого из них по отдельности, поэтому общее сопротивление ниже.

    Каждый резистор, включенный параллельно, имеет такое же полное напряжение источника, как и он, но делит общий ток между ними. Примером может служить соединение двух лампочек в параллельную цепь с аккумулятором на 1,5 В. В последовательной цепи две лампочки будут вдвое менее тусклыми при подключении к одному источнику батареи. Однако, если бы две лампочки были подключены параллельно, они были бы столь же яркими, как если бы они были подключены к батарее по отдельности. Поскольку к обеим лампочкам подается одинаковое полное напряжение, батарея также разряжается быстрее, поскольку она по существу обеспечивает полную энергию для обеих лампочек.В последовательной цепи батарея будет работать столько же, сколько и с одной лампочкой, только тогда яркость распределяется между лампочками.

    Комбинированные схемы

    Комбинированная схема может быть разбита на аналогичные части, которые работают последовательно или параллельно.

    Цели обучения

    Описать расположение резисторов в комбинированной схеме и его практическое значение

    Основные выводы

    Ключевые моменты
    • Более сложные соединения резисторов иногда представляют собой просто комбинации последовательного и параллельного.
    • Различные части комбинированной схемы могут быть идентифицированы как последовательные или параллельные, уменьшены до их эквивалентов, а затем уменьшены до тех пор, пока не останется единственное сопротивление.
    • Сопротивление в проводах снижает ток и мощность, подаваемые на резистор. Если сопротивление в проводах относительно велико, как в изношенном (или очень длинном) удлинителе, то эти потери могут быть значительными и повлиять на выходную мощность в бытовые приборы.
    Ключевые термины
    • серия : ряд элементов, которые следуют одно за другим или связаны друг за другом.
    • параллельно : Расположение электрических компонентов, при котором ток течет по двум или более путям.
    • Комбинированная схема : электрическая цепь, содержащая несколько резисторов, которые соединены как последовательным, так и параллельным соединением.

    Комбинированные схемы

    Более сложные соединения резисторов иногда представляют собой просто комбинации последовательного и параллельного. Это часто встречается, особенно если учитывать сопротивление проводов.В этом случае сопротивление провода включено последовательно с другими сопротивлениями, включенными параллельно.

    Комбинированная цепь может быть разбита на аналогичные части, которые являются последовательными или параллельными, как показано на схеме. На рисунке общее сопротивление может быть вычислено путем соединения трех резисторов друг с другом последовательно или параллельно. R 1 и R 2 соединены параллельно по отношению друг к другу, поэтому мы знаем, что для этого подмножества сопротивление, обратное сопротивлению, будет равно:

    Сеть резисторов : В этой комбинированной схеме цепь может быть разбита на последовательный компонент и параллельный компонент.

    Комбинированные схемы : Два параллельных резистора, соединенные последовательно с одним резистором.

    [латекс] \ frac {1} {\ text {R} _1} + \ frac {1} {\ text {R} _2} [/ latex] или [латекс] \ frac {\ text {R} _1 \ text {R} _2} {\ text {R} _1 + \ text {R} _2} [/ latex]

    R 3 соединены последовательно с и R 1 и R 2 , поэтому сопротивление будет рассчитано как:

    [латекс] \ text {R} = \ frac {\ text {R} _1 \ text {R} _2} {\ text {R} _1 + \ text {R} _2} + \ text {R} _3 [/ latex ]

    Сложные комбинированные схемы

    Для более сложных комбинированных схем различные части могут быть идентифицированы как последовательные или параллельные, уменьшены до их эквивалентов, а затем уменьшены до тех пор, пока не останется единственное сопротивление, как показано на.На этом рисунке комбинация из семи резисторов была идентифицирована как включенные последовательно или параллельно. На исходном изображении две обведенные кружком секции показывают резисторы, включенные параллельно.

    Уменьшение комбинированной схемы : Эта комбинация из семи резисторов имеет как последовательные, так и параллельные части. Каждый из них идентифицируется и приводится к эквивалентному сопротивлению, а затем уменьшается до тех пор, пока не будет достигнуто одно эквивалентное сопротивление.

    Уменьшение этих параллельных резисторов до одного значения R позволяет нам визуализировать схему в более упрощенном виде.На верхнем правом изображении мы видим, что обведенная кружком часть содержит два последовательно соединенных резистора. Мы можем дополнительно уменьшить это до другого значения R, добавив их. Следующий шаг показывает, что два обведенных резистора включены параллельно. Уменьшение тех бликов, что последние два соединены последовательно и, таким образом, могут быть уменьшены до одного значения сопротивления для всей цепи.

    Одним из практических следствий комбинированной схемы является то, что сопротивление в проводах снижает ток и мощность, подаваемую на резистор.Комбинированная цепь может быть преобразована в последовательную цепь на основе понимания эквивалентного сопротивления параллельных ветвей комбинированной цепи. Последовательная цепь может использоваться для определения общего сопротивления цепи. По сути, сопротивление провода является последовательным с резистором. Таким образом, увеличивается общее сопротивление и уменьшается ток. Если сопротивление провода относительно велико, как в изношенном (или очень длинном) удлинителе, то эти потери могут быть значительными. Если потребляется большой ток, падение ИК-излучения в проводах также может быть значительным.

    Зарядка аккумулятора: последовательные и параллельные ЭДС

    При последовательном включении источников напряжения их ЭДС и внутренние сопротивления складываются; параллельно они остаются прежними.

    Цели обучения

    Сравнить сопротивления и электродвижущие силы для источников напряжения, подключенных с одинаковой и противоположной полярностью, последовательно и параллельно

    Основные выводы

    Ключевые моменты
    • ЭДС, соединенные последовательно с одинаковой полярностью, являются аддитивными и приводят к более высокой общей ЭДС.
    • Две ЭДС, соединенные последовательно с противоположной полярностью, имеют общую ЭДС, равную разнице между ними, и могут использоваться для зарядки источника более низкого напряжения.
    • Два источника напряжения с идентичными ЭДС, соединенные параллельно, имеют чистую ЭДС, эквивалентную одному источнику ЭДС, однако чистое внутреннее сопротивление меньше и, следовательно, дает более высокий ток.
    Ключевые термины
    • параллельно : Расположение электрических компонентов, при котором ток течет по двум или более путям.
    • электродвижущая сила : (ЭДС) – напряжение, генерируемое батареей или магнитной силой в соответствии с законом Фарадея. Она измеряется в вольтах, а не в ньютонах, и поэтому на самом деле не является силой.
    • серия : ряд элементов, которые следуют одно за другим или связаны друг за другом.

    Когда используется более одного источника напряжения, они могут быть подключены последовательно или параллельно, аналогично резисторам в цепи.Когда источники напряжения включены последовательно в одном направлении, их внутренние сопротивления складываются, а их электродвижущая сила или ЭДС складываются алгебраически. Эти типы источников напряжения распространены в фонариках, игрушках и других приборах. Обычно ячейки включены последовательно, чтобы обеспечить большую суммарную ЭДС.

    Фонарик и лампочка : Последовательное соединение двух источников напряжения в одном направлении. Эта схема представляет собой фонарик с двумя последовательно включенными ячейками (источниками напряжения) и одной лампочкой (сопротивление нагрузки).

    Батарея – это соединение нескольких гальванических элементов. Однако недостатком такого последовательного соединения ячеек является то, что их внутреннее сопротивление увеличивается. Иногда это может быть проблематично. Например, если вы поместите в машину две батареи на 6 В вместо обычной батареи на 12 В, вы должны добавить как ЭДС, так и внутреннее сопротивление каждой батареи. Таким образом, вы получите ту же ЭДС 12 В, хотя внутреннее сопротивление тогда будет удвоено, что вызовет у вас проблемы, когда вы захотите запустить двигатель.

    Но, если ячейки противостоят друг другу – например, когда одна вставляется в прибор задом наперед, – общая ЭДС меньше, так как это алгебраическая сумма отдельных ЭДС. Когда он перевернут, он создает ЭДС, которая противодействует другой, и приводит к разнице между двумя источниками напряжения.

    Зарядное устройство : представляет собой два источника напряжения, соединенных последовательно с противоположными ЭДС. Ток течет в направлении большей ЭДС и ограничивается суммой внутренних сопротивлений.(Обратите внимание, что каждая ЭДС представлена ​​на рисунке буквой E.) Зарядное устройство, подключенное к аккумулятору, является примером такого подключения. Зарядное устройство должно иметь большую ЭДС, чем батарея, чтобы через него протекал обратный ток.

    Когда два источника напряжения с идентичными ЭДС соединены параллельно и также подключены к сопротивлению нагрузки, общая ЭДС равна индивидуальным ЭДС. Но общее внутреннее сопротивление уменьшается, поскольку внутренние сопротивления параллельны. Таким образом, параллельное соединение может производить больший ток.

    Две идентичные ЭДС : Два источника напряжения с одинаковыми ЭДС (каждый помечен буквой E), подключенные параллельно, создают одинаковую ЭДС, но имеют меньшее общее внутреннее сопротивление, чем отдельные источники. Параллельные комбинации часто используются для подачи большего тока.

    ЭДС и напряжение на клеммах

    Выходное напряжение или напряжение на клеммах источника напряжения, такого как батарея, зависит от его электродвижущей силы и внутреннего сопротивления.

    Цели обучения

    Выразите взаимосвязь между электродвижущей силой и напряжением на клеммах в форме уравнения

    Основные выводы

    Ключевые моменты
    • Электродвижущая сила (ЭДС) – это разность потенциалов источника при отсутствии тока.
    • Напряжение на клеммах – это выходное напряжение устройства, измеренное на его клеммах.
    • Напряжение на клеммах рассчитывается по формуле V = ЭДС – Ir.
    Ключевые термины
    • электродвижущая сила : (ЭДС) – напряжение, генерируемое батареей или магнитной силой в соответствии с законом Фарадея. Она измеряется в вольтах, а не в ньютонах, и поэтому на самом деле не является силой.
    • напряжение на клеммах : выходное напряжение устройства, измеренное на его клеммах.
    • разность потенциалов : разница в потенциальной энергии между двумя точками в электрическом поле; разница в заряде между двумя точками в электрической цепи; Напряжение.

    Когда вы забываете выключить автомобильные фары, они постепенно тускнеют по мере разрядки аккумулятора. Почему они просто не мигают, когда батарея разряжена? Их постепенное затемнение означает, что выходное напряжение батареи уменьшается по мере разряда батареи. Причина снижения выходного напряжения для разряженных или перегруженных батарей заключается в том, что все источники напряжения состоят из двух основных частей – источника электрической энергии и внутреннего сопротивления.

    Электродвижущая сила

    Все источники напряжения создают разность потенциалов и могут подавать ток, если подключены к сопротивлению. В небольшом масштабе разность потенциалов создает электрическое поле, которое воздействует на заряды, вызывая ток. Мы называем эту разность потенциалов электродвижущей силой (сокращенно ЭДС). ЭДС – это вообще не сила; это особый тип разности потенциалов источника при отсутствии тока. Единицы измерения ЭДС – вольты.

    Электродвижущая сила напрямую связана с источником разности потенциалов, например, с конкретной комбинацией химических веществ в батарее.Однако при протекании тока ЭДС отличается от выходного напряжения устройства. Напряжение на выводах батареи, например, меньше, чем ЭДС, когда батарея подает ток, и оно падает дальше, когда батарея разряжается или разряжается. Однако, если выходное напряжение устройства можно измерить без потребления тока, то выходное напряжение будет равно ЭДС (даже для сильно разряженной батареи).

    Напряжение на клеммах

    представляет схематическое изображение источника напряжения.Выходное напряжение устройства измеряется на его клеммах и называется напряжением на клеммах В, . Напряжение на клеммах определяется уравнением:

    Схематическое изображение источника напряжения : Любой источник напряжения (в данном случае углеродно-цинковый сухой элемент) имеет ЭДС, связанную с источником разности потенциалов, и внутреннее сопротивление r, связанное с его конструкцией. (Обратите внимание, что сценарий E означает ЭДС.) Также показаны выходные клеммы, на которых измеряется напряжение на клеммах V.Поскольку V = ЭДС-Ir, напряжение на клеммах равно ЭДС, только если ток не течет.

    [латекс] \ text {V} = \ text {emf} – \ text {Ir} [/ latex],

    где r – внутреннее сопротивление, а I – ток, протекающий во время измерения.

    I является положительным, если ток течет от положительного вывода. Чем больше ток, тем меньше напряжение на клеммах. Точно так же верно, что чем больше внутреннее сопротивление, тем меньше напряжение на клеммах.

    резисторов последовательно и параллельно

    Цели обучения

    К концу этого раздела вы сможете:

    • Нарисуйте цепь с резисторами, включенными параллельно и последовательно.
    • Рассчитайте падение напряжения тока на резисторе, используя закон Ома.
    • Contrast Способ расчета общего сопротивления для резисторов, включенных последовательно и параллельно.
    • Объясните, почему полное сопротивление параллельной цепи меньше наименьшего сопротивления любого из резисторов в этой цепи.
    • Вычислите общее сопротивление цепи, которая содержит смесь резисторов, соединенных последовательно и параллельно.

    Большинство схем имеет более одного компонента, называемого резистором , который ограничивает поток заряда в цепи.Мера этого предела расхода заряда называется сопротивлением . Простейшие комбинации резисторов – это последовательное и параллельное соединение, показанное на рисунке 1. Общее сопротивление комбинации резисторов зависит как от их индивидуальных значений, так и от способа их подключения.

    Рис. 1. (a) Последовательное соединение резисторов. (б) Параллельное соединение резисторов.

    Когда резисторы в серии ? Резисторы включены последовательно всякий раз, когда поток заряда, называемый током , должен проходить через устройства последовательно.Например, если ток течет через человека, держащего отвертку, и попадает в Землю, тогда R 1 на Рисунке 1 (а) может быть сопротивлением вала отвертки, R 2 сопротивлением ее ручки , R 3 сопротивление тела человека и R 4 сопротивление его обуви. На рисунке 2 показаны резисторы, последовательно подключенные к источнику напряжения . Кажется разумным, что полное сопротивление является суммой отдельных сопротивлений, учитывая, что ток должен проходить через каждый резистор последовательно.(Этот факт был бы преимуществом для человека, желающего избежать поражения электрическим током, который мог бы уменьшить ток, надев обувь с резиновыми подошвами с высоким сопротивлением. прибор, уменьшающий рабочий ток.) ​​

    Рис. 2. Три резистора, подключенных последовательно к батарее (слева), и эквивалентное одиночное или последовательное сопротивление (справа).

    Чтобы убедиться, что последовательно включенные сопротивления действительно складываются, давайте рассмотрим потерю электроэнергии, называемую падением напряжения , в каждом резисторе на Рисунке 2.Согласно закону Ома падение напряжения на резисторе В, , когда через него протекает ток, рассчитывается по формуле В = IR , где I равно току в амперах (А) и R – сопротивление в Ом (Ом). Другой способ представить это: В, – это напряжение, необходимое для протекания тока I через сопротивление R . Таким образом, падение напряжения на R 1 составляет В 1 = IR 1 , что на R 2 составляет В 2 = IR 2 и что для R 3 это V 3 = IR 3 .Сумма этих напряжений равна выходному напряжению источника; то есть

    В = В 1 + В 2 + В 3 .

    Это уравнение основано на сохранении энергии и сохранении заряда. Электрическая потенциальная энергия может быть описана уравнением PE = qV , где q – электрический заряд, а В, – напряжение. Таким образом, энергия, подводимая источником, составляет кв / , а рассеиваемая резисторами –

    .

    qV 1 + qV 2 + qV 3 .

    Установление связей: законы сохранения

    Вывод выражений для последовательного и параллельного сопротивления основан на законах сохранения энергии и сохранения заряда, которые утверждают, что общий заряд и полная энергия постоянны в любом процессе. Эти два закона непосредственно участвуют во всех электрических явлениях и будут многократно использоваться для объяснения как конкретных эффектов, так и общего поведения электричества.

    Эти энергии должны быть равны, потому что в цепи нет другого источника и другого назначения для энергии.Таким образом, qV = qV 1 + qV 2 + qV 3 . Плата q аннулируется, давая V = V 1 + V 2 + V 3 , как указано. (Обратите внимание, что одинаковое количество заряда проходит через батарею и каждый резистор за заданный промежуток времени, поскольку нет емкости для хранения заряда, нет места для утечки заряда и заряд сохраняется.) Теперь подстановка значений для отдельных напряжений дает

    В = IR 1 + IR 2 + IR 3 = I ( R 1 + R 2 + R 3 ).

    Обратите внимание, что для эквивалентного сопротивления одной серии R с , мы имеем

    В = ИК с .

    Это означает, что полное или эквивалентное последовательное сопротивление R с трех резисторов составляет R с = R 1 + R 2 + R 3 .Эта логика действительна в общем для любого количества резисторов, включенных последовательно; таким образом, полное сопротивление R с последовательного соединения составляет

    R с = R 1 + R 2 + R 3 +…,

    , как предлагается. Поскольку весь ток должен проходить через каждый резистор, он испытывает сопротивление каждого, а последовательно соединенные сопротивления просто складываются.

    Пример 1. Расчет сопротивления, тока, падения напряжения и рассеиваемой мощности: анализ последовательной цепи

    Предположим, что выходное напряжение батареи на рисунке 2 равно 12.0 В, а сопротивления равны R 1 = 1,00 Ом, R 2 = 6,00 Ом и R 3 = 13,0 Ом. а) Каково полное сопротивление? (б) Найдите ток. (c) Рассчитайте падение напряжения на каждом резисторе и покажите, как они складываются, чтобы равняться выходному напряжению источника. (d) Рассчитайте мощность, рассеиваемую каждым резистором. (e) Найдите выходную мощность источника и покажите, что она равна общей мощности, рассеиваемой резисторами.

    Стратегия и решение для (а)

    Общее сопротивление – это просто сумма отдельных сопротивлений, определяемая следующим уравнением:

    [латекс] \ begin {array} {lll} {R} _ {\ text {s}} & = & {R} _ {1} + {R} _ {2} + {R} _ {3} \ \ & = & 1.00 \ text {} \ Omega + 6.00 \ text {} \ Omega + 13.0 \ text {} \ Omega \\ & = & 20.0 \ text {} \ Omega \ end {array} \\ [/ latex].

    Стратегия и решение для (b)

    Ток определяется по закону Ома: В = IR . Ввод значения приложенного напряжения и общего сопротивления дает ток для цепи:

    [латекс] I = \ frac {V} {{R} _ {\ text {s}}} = \ frac {12.0 \ text {V}} {20.0 \ text {} \ Omega} = 0.60 \ text {A }\\[/латекс].

    Стратегия и решение для (c)

    Напряжение – или падение IR – на резисторе определяется законом Ома.Ввод значения тока и значения первого сопротивления дает

    .

    В 1 = IR 1 = (0,600 A) (1,0 Ом) = 0,600 В.

    Аналогично

    В 2 = IR 2 = (0,600 A) (6,0 Ом) = 3,60 В

    и

    V3 = IR 3 = (0,600 A) (13,0 Ом) = 7,80 В.

    Обсуждение для (c)

    Три капли IR добавляют к 12.0 В, прогноз:

    В 1 + В 2 + В 3 = (0,600 + 3,60 + 7,80) В = 12,0 В.

    Стратегия и решение для (d)

    Самый простой способ рассчитать мощность в ваттах (Вт), рассеиваемую резистором в цепи постоянного тока, – это использовать закон Джоуля , P = IV , где P – электрическая мощность. В этом случае через каждый резистор протекает одинаковый полный ток.Подставляя закон Ома В = IR в закон Джоуля, мы получаем мощность, рассеиваемую первым резистором, как

    .

    P 1 = I 2 R 1 = (0,600 A) 2 (1,00 Ом) = 0,360 Вт

    Аналогично

    P 2 = I 2 R 2 = (0,600 A) 2 (6,00 Ом) = 2,16 Вт.

    и

    P 3 = I 2 R 3 = (0.{2}} {R} \\ [/ latex], где В, – это падение напряжения на резисторе (а не полное напряжение источника). Будут получены те же значения.

    Стратегия и решение для (e)

    Самый простой способ рассчитать выходную мощность источника – использовать P = IV , где В, – напряжение источника. Это дает

    P = (0,600 A) (12,0 В) = 7,20 Вт.

    Обсуждение для (e)

    Обратите внимание, что по совпадению общая мощность, рассеиваемая резисторами, также равна 7.20 Вт, столько же, сколько мощность, выдаваемая источником. То есть

    P 1 + P 2 + P 3 = (0,360 + 2,16 + 4,68) W = 7,20 Вт.

    Мощность – это энергия в единицу времени (ватт), поэтому для сохранения энергии требуется, чтобы выходная мощность источника была равна общей мощности, рассеиваемой резисторами.

    Основные характеристики резисторов серии

    1. Последовательные сопротивления добавить: R с = R 1 + R 2 + R 3 +….
    2. Одинаковый ток протекает последовательно через каждый резистор.
    3. Отдельные последовательно включенные резисторы не получают полное напряжение источника, а делят его.

    На рис. 3 показаны резисторы параллельно , подключенные к источнику напряжения. Резисторы включены параллельно, когда каждый резистор подключен непосредственно к источнику напряжения с помощью соединительных проводов с незначительным сопротивлением. Таким образом, к каждому резистору приложено полное напряжение источника. Каждый резистор потребляет такой же ток, как если бы он один был подключен к источнику напряжения (при условии, что источник напряжения не перегружен).Например, автомобильные фары, радио и т. Д. Подключены параллельно, так что они используют полное напряжение источника и могут работать полностью независимо. То же самое и в вашем доме, или в любом другом здании. (См. Рисунок 3 (b).)

    Рис. 3. (a) Три резистора, подключенных параллельно батарее, и эквивалентное одиночное или параллельное сопротивление. (б) Электроснабжение в доме. (Источник: Dmitry G, Wikimedia Commons)

    Чтобы найти выражение для эквивалентного параллельного сопротивления R p , давайте рассмотрим протекающие токи и их связь с сопротивлением.Поскольку каждый резистор в цепи имеет полное напряжение, токи, протекающие через отдельные резисторы, равны [латекс] {I} _ {1} = \ frac {V} {{R} _ {1}} \\ [/ latex] , [латекс] {I} _ {2} = \ frac {V} {{R} _ {2}} \\ [/ latex] и [латекс] {I} _ {3} = \ frac {V} {{R} _ {3}} \\ [/ латекс]. Сохранение заряда подразумевает, что полный ток I , производимый источником, является суммой этих токов:

    I = I 1 + I 2 + I 3 .

    Подстановка выражений для отдельных токов дает

    [латекс] I = \ frac {V} {{R} _ {1}} + \ frac {V} {{R} _ {2}} + \ frac {V} {{R} _ {3}} = V \ left (\ frac {1} {{R} _ {1}} + \ frac {1} {{R} _ {2}} + \ frac {1} {{R} _ {3}} \ справа) \\ [/ латекс].

    Обратите внимание, что закон Ома для эквивалентного одиночного сопротивления дает

    [латекс] I = \ frac {V} {{R} _ {p}} = V \ left (\ frac {1} {{R} _ {p}} \ right) \\ [/ latex].

    Члены в скобках в последних двух уравнениях должны быть равны. Обобщая для любого количества резисторов, общее сопротивление R p параллельного соединения связано с отдельными сопротивлениями на

    .

    [латекс] \ frac {1} {{R} _ {p}} = \ frac {1} {{R} _ {1}} + \ frac {1} {{R} _ {2}} + \ гидроразрыв {1} {{R} _ {\ text {.} 3}} + \ text {.} \ Text {…} \\ [/ latex]

    Это соотношение приводит к общему сопротивлению R p , которое меньше наименьшего из отдельных сопротивлений. (Это видно в следующем примере.) При параллельном подключении резисторов от источника течет больше тока, чем протекает по любому из них по отдельности, поэтому общее сопротивление ниже.

    Пример 2. Расчет сопротивления, тока, рассеиваемой мощности и выходной мощности: анализ параллельной цепи

    Пусть выходное напряжение батареи и сопротивления в параллельном соединении на Рисунке 3 будут такими же, как и в ранее рассмотренном последовательном соединении: В = 12.0 В, R 1 = 1,00 Ом, R 2 = 6,00 Ом и R 3 = 13,0 Ом. а) Каково полное сопротивление? (б) Найдите полный ток. (c) Рассчитайте токи в каждом резисторе и покажите, как они складываются, чтобы равняться общему выходному току источника. (d) Рассчитайте мощность, рассеиваемую каждым резистором. (e) Найдите выходную мощность источника и покажите, что она равна общей мощности, рассеиваемой резисторами.

    Стратегия и решение для (а)

    Общее сопротивление для параллельной комбинации резисторов находится с помощью следующего уравнения.Ввод известных значений дает

    [латекс] \ frac {1} {{R} _ {p}} = \ frac {1} {{R} _ {1}} + \ frac {1} {{R} _ {2}} + \ frac {1} {{R} _ {3}} = \ frac {1} {1 \ text {.} \ text {00} \ text {} \ Omega} + \ frac {1} {6 \ text {. } \ text {00} \ text {} \ Omega} + \ frac {1} {\ text {13} \ text {.} 0 \ text {} \ Omega} \\ [/ latex].

    Таким образом,

    [латекс] \ frac {1} {{R} _ {p}} = \ frac {1.00} {\ text {} \ Omega} + \ frac {0 \ text {.} \ Text {1667}} {\ текст {} \ Omega} + \ frac {0 \ text {.} \ text {07692}} {\ text {} \ Omega} = \ frac {1 \ text {.} \ text {2436}} {\ text { } \ Omega} \\ [/ латекс].

    (Обратите внимание, что в этих расчетах каждый промежуточный ответ отображается с дополнительной цифрой.) Мы должны перевернуть это, чтобы найти полное сопротивление R p . Это дает

    [латекс] {R} _ {\ text {p}} = \ frac {1} {1 \ text {.} \ Text {2436}} \ text {} \ Omega = 0 \ text {.} \ Text { 8041} \ text {} \ Omega \\ [/ latex].

    Суммарное сопротивление с правильным количеством значащих цифр составляет R p = 0,804 Ом

    Обсуждение для (а)

    R p , как и предполагалось, меньше наименьшего индивидуального сопротивления.

    Стратегия и решение для (b)

    Полный ток можно найти из закона Ома, заменив полное сопротивление R p . Это дает

    [латекс] I = \ frac {V} {{R} _ {\ text {p}}} = \ frac {\ text {12.0 V}} {0.8041 \ text {} \ Omega} = \ text {14} \ text {.} \ text {92 A} \\ [/ latex].

    Обсуждение для (б)

    Ток I для каждого устройства намного больше, чем для тех же устройств, подключенных последовательно (см. Предыдущий пример).Схема с параллельным соединением имеет меньшее общее сопротивление, чем резисторы, включенные последовательно.

    Стратегия и решение для (c)

    Отдельные токи легко вычислить по закону Ома, поскольку каждый резистор получает полное напряжение. Таким образом,

    [латекс] {I} _ {1} = \ frac {V} {{R} _ {1}} = \ frac {12.0 \ text {V}} {1.00 \ text {} \ Omega} = 12.0 \ text {A} \\ [/ латекс].

    Аналогично

    [латекс] {I} _ {2} = \ frac {V} {{R} _ {2}} = \ frac {12.0 \ text {V}} {6.00 \ text {} \ Omega} = 2 \ text {.} \ text {00} \ text {A} \\ [/ latex]

    и

    [латекс] {I} _ {3} = \ frac {V} {{R} _ {3}} = \ frac {\ text {12} \ text {.} 0 \ text {V}} {\ text {13} \ text {.} \ Text {0} \ text {} \ Omega} = 0 \ text {.} \ Text {92} \ text {A} \\ [/ latex].

    Обсуждение для (c)

    Общий ток складывается из отдельных токов:

    I 1 + I 2 + I 3 = 14,92 A.

    Это соответствует сохранению заряда.{2}} {13.0 \ text {} \ Omega} = 11.1 \ text {W} \\ [/ latex].

    Обсуждение для (d)

    Мощность, рассеиваемая каждым резистором параллельно, значительно выше, чем при последовательном подключении к тому же источнику напряжения.

    Стратегия и решение для (e)

    Общую мощность также можно рассчитать несколькими способами. Выбрав P = IV и введя полный ток, получим

    P = IV = (14,92 A) (12,0 В) = 179 Вт.

    Обсуждение для (e)

    Суммарная мощность, рассеиваемая резисторами, также 179 Вт:

    P 1 + P 2 + P 3 = 144 Вт + 24,0 Вт + 11,1 Вт = 179 Вт

    Это соответствует закону сохранения энергии.

    Общее обсуждение

    Обратите внимание, что как токи, так и мощность при параллельном подключении больше, чем для тех же устройств, подключенных последовательно.

    Основные характеристики параллельных резисторов
    1. Параллельное сопротивление определяется из [latex] \ frac {1} {{R} _ {\ text {p}}} = \ frac {1} {{R} _ {1}} + \ frac {1} { {R} _ {2}} + \ frac {1} {{R} _ {3}} + \ text {…} \\ [/ latex], и оно меньше любого отдельного сопротивления в комбинации.
    2. На каждый параллельно включенный резистор подается такое же полное напряжение источника. (В системах распределения электроэнергии чаще всего используются параллельные соединения для питания бесчисленных устройств, обслуживаемых одним и тем же напряжением, и для того, чтобы они могли работать независимо.)
    3. Параллельные резисторы не получают суммарный ток каждый; они делят это.

    Сочетания последовательного и параллельного

    Более сложные соединения резисторов иногда представляют собой просто комбинации последовательного и параллельного. Они часто встречаются, особенно если учесть сопротивление провода. В этом случае сопротивление провода включено последовательно с другими сопротивлениями, включенными параллельно. Комбинации последовательного и параллельного подключения можно свести к одному эквивалентному сопротивлению, используя методику, показанную на рисунке 4.Различные части идентифицируются как последовательные или параллельные, уменьшаются до их эквивалентов и далее уменьшаются до тех пор, пока не останется единственное сопротивление. Процесс занимает больше времени, чем труден.

    Рис. 4. Эта комбинация из семи резисторов имеет как последовательные, так и параллельные части. Каждый из них идентифицируется и приводится к эквивалентному сопротивлению, а затем уменьшается до тех пор, пока не будет достигнуто одно эквивалентное сопротивление.

    Самая простая комбинация последовательного и параллельного сопротивления, показанная на рисунке 4, также является наиболее поучительной, поскольку она используется во многих приложениях.Например, R 1 может быть сопротивлением проводов от автомобильного аккумулятора к его электрическим устройствам, которые подключены параллельно. R 2 и R 3 могли быть стартером и светом салона. Ранее мы предполагали, что сопротивление провода незначительно, но, когда это не так, оно имеет важные последствия, как показывает следующий пример.

    Пример 3. Расчет сопротивления,

    IR Падение, ток и рассеиваемая мощность: объединение последовательных и параллельных цепей

    На рис. 5 показаны резисторы из двух предыдущих примеров, подключенные другим способом – комбинацией последовательного и параллельного подключения.Мы можем считать R 1 сопротивлением проводов, ведущих к R 2 и R 3 . (а) Найдите полное сопротивление. (b) Что такое падение IR в R 1 ? (c) Найдите текущие значения от I 2 до R 2 . (d) Какую мощность рассеивает R 2 ?

    Рис. 5. Эти три резистора подключены к источнику напряжения, так что R 2 и R 3 параллельны друг другу, и эта комбинация включена последовательно с R 1 .

    Стратегия и решение для (а)

    Чтобы найти полное сопротивление, отметим, что R 2 и R 3 находятся параллельно, а их комбинация R p находится последовательно с R 1 . Таким образом, полное (эквивалентное) сопротивление этой комбинации составляет

    .

    R итого = R 1 + R p .

    Сначала мы находим R p , используя уравнение для параллельных резисторов и вводя известные значения:

    [латекс] \ frac {1} {{R} _ {\ text {p}}} = \ frac {1} {{R} _ {2}} + \ frac {1} {{R} _ {3 }} = \ frac {1} {6 \ text {.} \ text {00} \ text {} \ Omega} + \ frac {1} {\ text {13} \ text {.} 0 \ text {} \ Omega} = \ frac {0.2436} {\ text {} \ Омега} \\ [/ латекс].

    Инвертирование дает

    [латекс] {R} _ {\ text {p}} = \ frac {1} {0,2436} \ text {} \ Omega = 4.11 \ text {} \ Omega \\ [/ latex].

    Таким образом, общее сопротивление равно

    .

    R всего = R 1 + R p = 1,00 Ом + 4,11 Ом = 5,11 Ом.

    Обсуждение для (а)

    Общее сопротивление этой комбинации является промежуточным между значениями чистой серии и чистой параллели (20.0 Ом и 0,804 Ом соответственно), найденные для тех же резисторов в двух предыдущих примерах.

    Стратегия и решение для (b)

    Чтобы найти падение IR в R 1 , отметим, что полный ток I протекает через R 1 . Таким образом, падение IR составляет

    .

    В 1 = ИК 1

    Мы должны найти I , прежде чем сможем вычислить V 1 .Полный ток I находится с помощью закона Ома для схемы. То есть

    [латекс] I = \ frac {V} {{R} _ {\ text {tot}}} = \ frac {\ text {12.0} \ text {V}} {5.11 \ text {} \ Omega} = 2.35 \ text {A} \\ [/ latex].

    Вводя это в выражение выше, мы получаем

    В 1 = IR 1 = (2,35 A) (1,00 Ом) = 2,35 В.

    Обсуждение для (б)

    Напряжение, приложенное к R 2 и R 3 , меньше полного напряжения на величину В 1 .Когда сопротивление провода велико, это может существенно повлиять на работу устройств, представленных R 2 и R 3 .

    Стратегия и решение для (c)

    Чтобы найти ток через R 2 , мы должны сначала найти приложенное к нему напряжение. Мы называем это напряжение В, , , , потому что оно приложено к параллельной комбинации резисторов. Напряжение, приложенное как к R 2 , так и к R 3 , уменьшается на величину В 1 , и поэтому оно составляет

    .

    В p = V V 1 = 12.0 В – 2,35 В = 9,65 В.

    Теперь ток I 2 через сопротивление R 2 находится по закону Ома:

    [латекс] {I} _ {2} = \ frac {{V} _ {\ text {p}}} {{R} _ {2}} = \ frac {9.65 \ text {V}} {6.00 \ текст {} \ Omega} = 1,61 \ text {A} \\ [/ latex].

    Обсуждение для (c)

    Ток меньше, чем 2,00 А, которые протекали через R 2 , когда он был подключен параллельно к батарее в предыдущем примере параллельной цепи.

    Стратегия и решение для (d)

    Мощность, рассеиваемая R 2 , определяется

    P 2 = ( I 2 ) 2 R 2 = (1,61 A) 2 (6,00 Ом) = 15,5 Вт

    Обсуждение для (d)

    Мощность меньше 24,0 Вт, рассеиваемых этим резистором при параллельном подключении к источнику 12,0 В.

    Одним из следствий этого последнего примера является то, что сопротивление в проводах снижает ток и мощность, подаваемую на резистор.Если сопротивление провода относительно велико, как в изношенном (или очень длинном) удлинителе, то эти потери могут быть значительными. Если потребляется большой ток, падение IR в проводах также может быть значительным.

    Например, когда вы роетесь в холодильнике и включается мотор, свет холодильника на мгновение гаснет. Точно так же вы можете увидеть тусклый свет в салоне, когда вы запускаете двигатель вашего автомобиля (хотя это может быть связано с сопротивлением внутри самой батареи).

    То, что происходит в этих сильноточных ситуациях, показано на рисунке 6. Устройство, обозначенное номером R 3 , имеет очень низкое сопротивление, поэтому при его включении протекает большой ток. Этот увеличенный ток вызывает большее падение IR в проводах, представленных R 1 , уменьшая напряжение на лампе (которое составляет R 2 ), которое затем заметно гаснет.

    Рис. 6. Почему гаснет свет при включении большого прибора? Ответ заключается в том, что большой ток, потребляемый двигателем прибора, вызывает значительное падение напряжения в проводах и снижает напряжение на свету.

    Проверьте свое понимание

    Можно ли любую произвольную комбинацию резисторов разбить на последовательную и параллельную? Посмотрите, сможете ли вы нарисовать принципиальную схему резисторов, которые нельзя разбить на комбинации последовательно и параллельно.

    Раствор Нет, есть много способов подключения резисторов, которые не являются комбинациями последовательного и параллельного, включая петли и переходы. В таких случаях правила Кирхгофа, которые будут включены в Правила Кирхгофа, позволят вам проанализировать схему.

    Стратегии решения проблем для последовательных и параллельных резисторов
    1. Нарисуйте четкую принципиальную схему, пометив все резисторы и источники напряжения. Этот шаг включает список известных проблем, поскольку они отмечены на вашей принципиальной схеме.
    2. Определите, что именно необходимо определить в проблеме (определите неизвестные). Письменный список полезен.
    3. Определите, включены ли резисторы последовательно, параллельно или в комбинации последовательно и параллельно.Изучите принципиальную схему, чтобы сделать эту оценку. Резисторы включены последовательно, если через них должен последовательно проходить один и тот же ток.
    4. Используйте соответствующий список основных функций для последовательных или параллельных подключений, чтобы найти неизвестные. Есть один список для серий, а другой – для параллелей. Если ваша проблема представляет собой комбинацию последовательного и параллельного соединения, уменьшайте ее поэтапно, рассматривая отдельные группы последовательных или параллельных соединений, как это сделано в этом модуле и примерах. Особое примечание: при обнаружении R необходимо проявлять осторожность.
    5. Проверьте, являются ли ответы разумными и последовательными. Единицы и числовые результаты должны быть разумными. Общее последовательное сопротивление должно быть больше, а общее параллельное сопротивление, например, должно быть меньше. Мощность должна быть больше для одних и тех же устройств, подключенных параллельно, по сравнению с последовательными и т. Д.

    Сводка раздела

    Концептуальные вопросы

    1. Переключатель имеет переменное сопротивление, которое почти равно нулю в замкнутом состоянии и очень велико в разомкнутом состоянии, и он включен последовательно с устройством, которым он управляет.Объясните влияние переключателя на рис. 7 на ток в разомкнутом и замкнутом состоянии.

    Рис. 7. Переключатель обычно включается последовательно с источником сопротивления и напряжения. В идеале переключатель имеет почти нулевое сопротивление в замкнутом состоянии, но имеет чрезвычайно большое сопротивление в разомкнутом состоянии. (Обратите внимание, что на этой диаграмме скрипт E представляет напряжение (или электродвижущую силу) батареи.)

    2. Какое напряжение на разомкнутом переключателе на Рисунке 7?

    3. На разомкнутом переключателе есть напряжение, как на Рисунке 7.Почему же тогда мощность, рассеиваемая разомкнутым переключателем, мала?

    4. Почему мощность, рассеиваемая замкнутым переключателем, как на Рисунке 7, мала?

    5. Студент в физической лаборатории по ошибке подключил электрическую лампочку, батарею и выключатель, как показано на рисунке 8. Объясните, почему лампочка горит, когда выключатель разомкнут, и гаснет, когда выключатель замкнут. (Не пытайтесь – батарея сильно разряжается!)

    Рис. 8. Ошибка подключения. Включите этот переключатель параллельно устройству, обозначенному [латекс] R [/ латекс].(Обратите внимание, что на этой диаграмме скрипт E представляет напряжение (или электродвижущую силу) батареи.)

    6. Зная, что сила удара зависит от величины тока, протекающего через ваше тело, вы бы предпочли, чтобы он был включен последовательно или параллельно с сопротивлением, таким как нагревательный элемент тостера, если он потрясен им? Объяснять.

    7. Были бы ваши фары тусклыми при запуске двигателя автомобиля, если бы провода в вашем автомобиле были сверхпроводниками? (Не пренебрегайте внутренним сопротивлением батареи.) Объяснять.

    8. Некоторые гирлянды праздничных огней соединены последовательно для экономии затрат на проводку. В старой версии использовались лампочки, которые при перегорании прерывали электрическое соединение, как открытый выключатель. Если одна такая лампочка перегорит, что случится с остальными? Если такая цепочка работает от 120 В и имеет 40 одинаковых лампочек, каково нормальное рабочее напряжение каждой? В более новых версиях используются лампы, которые при перегорании замыкаются накоротко, как замкнутый выключатель. Если одна такая лампочка перегорит, что случится с остальными? Если такая цепочка работает от 120 В и в ней осталось 39 идентичных лампочек, каково тогда рабочее напряжение каждой?

    9.Если две бытовые лампочки мощностью 60 и 100 Вт подключить последовательно к бытовой электросети, какая из них будет ярче? Объяснять.

    10. Предположим, вы проводите физическую лабораторию, в которой вас просят вставить резистор в цепь, но все прилагаемые резисторы имеют большее сопротивление, чем запрошенное значение. Как бы вы соединили доступные сопротивления, чтобы попытаться получить меньшее запрошенное значение?

    11. Перед Второй мировой войной некоторые радиостанции получали питание через «шнур сопротивления», который имел значительное сопротивление.Такой резистивный шнур снижает напряжение до желаемого уровня для ламп радиоприемника и т.п., и это экономит расходы на трансформатор. Объясните, почему шнуры сопротивления нагреваются и тратят энергию при включенном радио.

    12. У некоторых лампочек есть три уровня мощности (не включая ноль), полученные от нескольких нитей накала, которые индивидуально переключаются и соединяются параллельно. Какое минимальное количество нитей нити необходимо для трех режимов мощности?

    Задачи и упражнения

    Примечание. Можно считать, что данные, взятые из цифр, имеют точность до трех значащих цифр.

    1. (а) Каково сопротивление десяти последовательно соединенных резисторов сопротивлением 275 Ом? (б) Параллельно?

    2. (a) Каково сопротивление последовательно соединенных резисторов 1,00 × 10 2 Ом, 2,50 кОм и 4,00 кОм? (б) Параллельно?

    3. Какое наибольшее и наименьшее сопротивление можно получить, соединив резисторы на 36,0 Ом, 50,0 Ом и 700 Ом?

    4. Тостер на 1800 Вт, электрическая сковорода на 1400 Вт и лампа на 75 Вт подключены к одной розетке в цепи 15 А, 120 В.(Три устройства работают параллельно, если подключены к одной розетке.) а) Какой ток потребляет каждое устройство? (b) Перегорит ли эта комбинация предохранитель на 15 А?

    5. Фара мощностью 30,0 Вт и стартер мощностью 2,40 кВт обычно подключаются параллельно в систему на 12,0 В. Какую мощность потребляли бы одна фара и стартер при последовательном подключении к батарее 12,0 В? (Не обращайте внимания на любое другое сопротивление в цепи и любое изменение сопротивления в двух устройствах.)

    6.(a) Учитывая батарею на 48,0 В и резисторы на 24,0 и 96,0 Ом, найдите для каждого из них ток и мощность при последовательном соединении. (b) Повторите, когда сопротивления включены параллельно.

    7. Ссылаясь на пример комбинирования последовательных и параллельных цепей и рисунок 5, вычислите I 3 двумя следующими способами: (a) по известным значениям I и I 2 ; (б) используя закон Ома для R 3 . В обеих частях явно показано, как вы следуете шагам, описанным выше в стратегии решения проблем для последовательных и параллельных резисторов .

    Рис. 5. Эти три резистора подключены к источнику напряжения, так что R 2 и R 3 параллельны друг другу, и эта комбинация включена последовательно с R 1 .

    8. Ссылаясь на рисунок 5: (a) Вычислите P 3 и обратите внимание на его сравнение с P 3 , найденным в первых двух примерах задач в этом модуле. (b) Найдите полную мощность, отдаваемую источником, и сравните ее с суммой мощностей, рассеиваемых резисторами.

    9. См. Рисунок 6 и обсуждение затемнения света при включении тяжелого прибора. (a) Учитывая, что источник напряжения составляет 120 В, сопротивление провода составляет 0,400 Ом, а номинальная мощность лампы составляет 75,0 Вт, какая мощность будет рассеиваться лампой, если при включении двигателя через провода пройдет в общей сложности 15,0 А? Предположите незначительное изменение сопротивления лампы. б) Какая мощность потребляет двигатель?

    Рис. 6. Почему гаснет свет при включении большого прибора? Ответ заключается в том, что большой ток, потребляемый двигателем прибора, вызывает значительное падение напряжения в проводах и снижает напряжение на свету.

    10. Линия электропередачи на 240 кВ, имеющая 5,00 × 10 2 , подвешена к заземленным металлическим опорам с помощью керамических изоляторов, каждый из которых имеет сопротивление 1,00 × 10 9 Ом (рис. 9 (а)). Какое сопротивление на землю у 100 изоляторов? (b) Рассчитайте мощность, рассеиваемую 100 из них. (c) Какая доля мощности, переносимой линией, составляет это? Ясно покажите, как вы следуете шагам, описанным выше в стратегии решения проблем для последовательных и параллельных резисторов .

    Рис. 9. Высоковольтная (240 кВ) линия электропередачи 5,00 × 10 2 подвешена к заземленной металлической опоре электропередачи. Ряд керамических изоляторов обеспечивает сопротивление 1,00 × 10 9 Ом каждый.

    11. Покажите, что если два резистора R 1 и R 2 объединены, и один из них намного больше другого ( R 1 >> R 2 ): (a ) Их последовательное сопротивление почти равно большему сопротивлению R 1 .(б) Их параллельное сопротивление почти равно меньшему сопротивлению R 2 .

    12. Необоснованные результаты Два резистора, один из которых имеет сопротивление 145 Ом, подключены параллельно, чтобы получить общее сопротивление 150 Ом. а) Каково значение второго сопротивления? б) Что неразумного в этом результате? (c) Какие предположения необоснованны или непоследовательны?

    13. Необоснованные результаты Два резистора, один из которых имеет сопротивление 900 кОм, соединены последовательно, чтобы получить общее сопротивление 0.500 МОм. а) Каково значение второго сопротивления? б) Что неразумного в этом результате? (c) Какие предположения необоснованны или непоследовательны?

    Глоссарий

    серия:
    последовательность резисторов или других компонентов, включенных в цепь один за другим
    резистор:
    компонент, обеспечивающий сопротивление току, протекающему через электрическую цепь
    сопротивление:
    , вызывая потерю электроэнергии в цепи
    Закон Ома:
    соотношение между током, напряжением и сопротивлением в электрической цепи: В = IR
    напряжение:
    – электрическая потенциальная энергия на единицу заряда; электрическое давление, создаваемое источником питания, например батареей
    падение напряжения:
    потеря электроэнергии при прохождении тока через резистор, провод или другой компонент
    ток:
    поток заряда через электрическую цепь мимо заданной точки измерения
    Закон Джоуля:
    соотношение между потенциальной электрической мощностью, напряжением и сопротивлением в электрической цепи, определяемое следующим образом: [latex] {P} _ {e} = \ text {IV} [/ latex]
    параллельно:
    подключение резисторов или других компонентов в электрической цепи таким образом, что каждый компонент получает одинаковое напряжение от источника питания; часто изображается на диаграмме в виде лестницы, где каждый компонент находится на ступеньке лестницы

    Избранные решения проблем и упражнения

    1.(а) 2,75 кОм (б) 27,5 Ом

    3. (а) 786 Ом (б) 20,3 Ом

    5. 29,6 Вт

    7. (а) 0,74 А (б) 0,742 А

    9. (а) 60,8 Вт (б) 3,18 кВт

    11. (a) [латекс] \ begin {array} {} {R} _ {\ text {s}} = {R} _ {1} + {R} _ {2} \\ \ Rightarrow {R} _ {\ text {s}} \ приблизительно {R} _ {1} \ left ({R} _ {1} \ text {>>} {R} _ {2} \ right) \ end {array} \\ [/ латекс]

    (b) [латекс] \ frac {1} {{R} _ {p}} = \ frac {1} {{R} _ {1}} + \ frac {1} {{R} _ {2} } = \ frac {{R} _ {1} + {R} _ {2}} {{R} _ {1} {R} _ {2}} \\ [/ latex],

    , так что

    [латекс] \ begin {array} {} {R} _ {p} = \ frac {{R} _ {1} {R} _ {2}} {{R} _ {1} + {R} _ {2}} \ приблизительно \ frac {{R} _ {1} {R} _ {2}} {{R} _ {1}} = {R} _ {2} \ left ({R} _ {1 } \ text {>>} {R} _ {2} \ right) \ text {.} \ end {array} \\ [/ latex]

    13. (a) –400 кОм (b) Сопротивление не может быть отрицательным. (c) Считается, что последовательное сопротивление меньше, чем у одного из резисторов, но должно быть больше, чем у любого из резисторов.

    Электрическое сопротивление в последовательных и параллельных сетях

    Последовательное соединение

    Общее сопротивление для резисторов, подключенных последовательно, можно рассчитать как

    R = R 1 + R 2 + …. + R n (1)

    где

    R = сопротивление (Ом, Ом)

    Пример – Резисторы в серии

    Подключены три резистора 33 Ом , 33 Ом и 47 Ом в сериале.Общее сопротивление можно рассчитать как

    R = ( 33 Ом) + ( 33 Ом) + ( 47 Ом)

    = 113 Ом

    Параллельное соединение

    Общее сопротивление для резисторов, подключенных параллельно, можно рассчитать как

    1 / R = 1 / R 1 + 1 / R 2 + …. + 1 / R n (2)

    Эквивалентное сопротивление 2 резисторов, соединенных параллельно, можно выразить как

    R = R 1 R 2 / (R 1 + R 2 ) (3)

    Пример – Параллельно резисторы

    Три резистора 33 Ом , 33 Ом и 47 Ом подключены параллельно.Общее сопротивление можно рассчитать как

    1 / R = 1 / ( 33 Ом ) + 1 / ( 33 Ом ) + 1 / (47 Ом )

    = 0,082 (1 / Ом)

    R = 1 / (0,082 Ом)

    = 12,2 Ом

    Если напряжение аккумулятора составляет 12 В – ток в цепи можно рассчитать по закону Ома

    I = U / R

    = (12 В) / (12.2 Ом)

    = 0,98 ампера

    Можно рассчитать ток через каждый резистор

    I 1 = U / R 1 = (12 В) / (33 Ом) = 0,36 ампера

    I 2 = U / R 2 = (12 В) / (33 Ом) = 0,36 ампера

    I 3 = U / R 3 = (12 В) / ( 47 Ом) = 0,26 ампера

    Параллельно подключенные резисторы – Калькулятор

    Сложите сопротивления до пяти параллельно подключенных резисторов и (необязательно) напряжение цепи.

    Общее сопротивление и ток – и отдельные токи во всех резисторах – будут рассчитаны:

    R 1 (Ом)

    R 2 (Ом)

    R 3 ( Ом)

    R 4 (Ом )

    R 5 (Ом)

    Напряжение (В)

    I 1 (амперы)

    I 2 (амперы)

    I 3 (амперы)

    I 4 (амперы)

    I 5 (амперы)

    R (Ом)

    I (амперы)

    .

    Добавить комментарий

    Ваш адрес email не будет опубликован. Обязательные поля помечены *